Vous êtes sur la page 1sur 80

iran

National Math Olympiad (3rd Round)


1997

2nd round

1 Let a, b, c be real numbers. Prove that there exists a triangle with side lengths a, b, c if and
only if 2(a4 + b4 + c4 ) > (a2 + b2 + c2 )2 .
2 Prove that if a, b, c, dare positive integers such that ad = bc, then a + b + c + dcannot be a
prime number.
3 Let N be the midpoint of side BC of triangle ABC. Right isosceles triangles ABM and ACP
are constructed outside the triangle, with bases AB and AC. Prove that 4M N P is also a
right isosceles triangle.
4 Let n blue points Ai and n red points Bi (i = 1, 2, ..., n) be situated on a line. Prove that
P
P
P
i,j Ai Bj
i<j Ai Aj +
i<j Bi Bj

This file was downloaded from the AoPS Math Olympiad Resources Page
http://www.artofproblemsolving.com/

Page 1

iran
National Math Olympiad (3rd Round)
1997

3rd round

1 Suppose that S is a finite set of real numbers with the property that any two distinct elements
of S form an arithmetic progression with another element inS. Give an example of such a set
with 5 elements and show that no such set exists with more than 5 elements.
2 Suppose that 10 points are given in the plane, such that among any five of them there are
four lying on a circle. Find the minimum number of these points which must lie on a circle.
3 Consider a circle with diameter AB and center O, and let C and D be two points on this
circle. The line CD meets the line AB at a point M satisfying M B < M A and M D < M C.
Let K be the point of intersection (different from O) of the circumcircles of triangles AOC
and DOB. Show that the lines M K and OK are perpendicular to each other, i. e. that
]M KO = 90 .
4 Determine all functions f : N0 N0 {1} such that for all n > 0 f (n + 1) + f (n + 3) =
f (n + 5)f (n + 7) 1375.
5 Let O be the circumcenter and H the orthocenter of an acute-angled triangle ABC such that
BC > CA. Let F be the foot of the altitude CH of triangle ABC. The perpendicular to the
line OF at the point F intersects the line AC at P . Prove that ]F HP = ]BAC.
6 LetA be a symmetric {0, 1}-matrix with all the diagonal entries equal to 1. Show that there
exist indices i1 < i2 < < ik nsuch that Ai1 + Ai2 + ... + Aik = (1, 1, ..., 1) ( mod 2)
where Ai denotes the i th column of A.

This file was downloaded from the AoPS Math Olympiad Resources Page
http://www.artofproblemsolving.com/

Page 2

iran
National Math Olympiad (3rd Round)
1997

4th round

1 Let n be a positive integer. Prove that there exist polynomialsf (x)and g(x) with integer
n
n
coefficients such that f (x)(x + 1)2 + g(x)(x2 + 1) = 2.
2 Suppose that f : R R has the following properties: (i) f (x) = 1 for all x; (ii) f (x + 13
42 ) +
1
1
f (x) = f (x + 6 ) + f (x + 7 ) for all x. Prove that f is periodic.
3 Let1 , 2 , ..., k be distinct real numbers with a nonzero sum. Prove that there exist integers
P
P
n1 , n2 , ..., nk such that ki=1 ni i > 0and for any non-identical permutation of {1, 2, . . . , k}, ki=1 ni (i) <
0
4 Let P be a variable point on arcBC of the circumcircle of triangle ABC not containing A.
Let I1 and I2 be the incenters of the triangles P AB and P AC respectively. Prove that: (a)
The circumcircle of ?P I1 I2 passes through a fixed point. (b) The circle with diameter I1 I2
passes through a fixed point. (c) The midpoint of I1 I2 lies on a fixed circle.
5 Suppose that f : R+ R+ is a function such that for all x, y > 0 f (x + y) + f (f (x) + f (y)) =
f (f (x + f (y)) + f (y + f (x))). Prove that f (x) = f 1 (x).
6 A building consists of finitely many rooms which have been separated by walls. There are
some doors on some of these walls which can be used to go around the building. Assume it
is possible to reach any room from any other room. Two fixed rooms are marked by S and
E. A person starts walking from S and wants to reachE.
A Program P = (Pi )iI is anR, L-sequence. The person uses it as follows: After passing
through the n-th door, he chooses the door just to the right or left from the door just passed,
meaning thatPn is R orL, and gets through it. In a room with one door, any symbol means
selecting the door he has just passed. The person stops as soon as he reaches E. Prove that
there is a (possibly infinite) program P with the property that, no matter how the structure
of the building is, the person can reach E by following it.

This file was downloaded from the AoPS Math Olympiad Resources Page
http://www.artofproblemsolving.com/

Page 3

iran
National Math Olympiad (3rd Round)
1998

2nd round

1 Let x and y be positive integers such that3x2 + x = 4y 2 + y. Prove that x yis a square.
2 Let KL and KN be tangent to the circle C (withL, N on C), and letM be a point on the
extension of KN beyond N . The circumcircle of triangle KLM meetsC again at P . PointQis
the foot of the perpendicular from N toM L. Prove that M P Q = 2KM L.
3 An n ntable is filled with numbers 1, 0, 1 in such a manner that every row and column
contains exactly one 1 and one 1. Prove that the rows and columns can be reordered so
that in the resulting table each number has been replaced with its negative.
P
P
P
4 Let x1 , x2 , x3 , x4 be positive numbers with the product 1. Prove that: 4i=1 x3i max{ 4i=1 xi , 4i=1 x1 }
5 In an acute triangle ABC,D is the foot of the altitude from A. The bisectors of the inner
angles B and C respectively meet AD atE and F . If BE = CF , prove that ABC is an
isosceles triangle.
q
6 Supposea, bare natural numbers such that: p = 4b 2ab
2a+b is a prime number. What is the
maximum possible value ofp?

This file was downloaded from the AoPS Math Olympiad Resources Page
http://www.artofproblemsolving.com/

Page 1

iran
National Math Olympiad (3rd Round)
1998

3rd round

1 Suppose thata, b, x are positive integers such that: xa+b = ab b prove that a = xand b = xx .
2 In an acute triangle ABC, points D, E, F are the feet of the altitudes from A, B, C, respectively. A line through D parallel to EF meetsAC at Q and AB at R. Lines BCand EF
intersect atP . Prove that the circumcircle of triangle P QR passes through the midpoint of
BC.
3 LetS = {x0 , x1 , . . . , xn } be a finite set of numbers in the interval [0, 1] withx0 = 0 and x1 = 1.
We consider pairwise distances between numbers in S. If every distance that appears, except
the distance 1, occurs at least twice, prove that all the xi are rational.
4 Let ABC and XY Z be two triangles. Define A1 = BC ZX, A2 = BC XY , B1 = CAXY ,
B2 = CA Y Z, C1 = AB Y Z, C2 = AB ZX. Hereby, the abbreviation g h means the
point of intersection of two lines g and h.
Prove that

C1 C2
AB

A1 A2
BC

B1 B2
CA

holds if and only if

A1 C2
XZ

C1 B 2
ZY

B1 A2
YX .

5 Let x, y, z be real numbers greater than 1 such that x1 + y1 + z1 = 2 Prove that: x 1+ y 1+

z1 x+y+z
6 LetP be the set of all points in Rn with rational coordinates. For points A, B P , one can
move from A to Bif the distance AB is 1. Prove that every point inP can be reached from
any other point in P by a finite sequence of moves if and only ifn 5.

This file was downloaded from the AoPS Math Olympiad Resources Page
http://www.artofproblemsolving.com/

Page 2

iran
National Math Olympiad (3rd Round)
1998

4th round

1 Let f1 , f2 , f3 : R R be functions such that a1 f1 + a2 f2 + a3 f3 is monotonous for all


a1 , a2 , a3 R. Show that there exist real numbers c1 , c2 , c3 , not all zero, such that c1 f1 (x) +
c2 f2 (x) + c3f 3(x) = 0 for all real x.
2 Let X be a set with n elements, and let A1 , A2 , ..., Am be subsets of X such that:
1) |Ai | = 3 for every i {1, 2, ..., m}; 2) |Ai Aj | 1 for all i, j {1, 2, ..., m} such that
i 6= j.

Prove that there exists a subset A of X such that A has at least 2n elements, and for every
i {1, 2, ..., m}, the set A does not contain Ai .
3 The edges of a regular 2n gon are colored red and blue in some fashion. A step consists
in recoloring each edge whose neighbors are both the same color in red, and recoloring each
edge whose neighbors are of opposite colors in blue. Prove that after 2n 1 steps all of the
edges will be red, and show that this neednt hold after fewer steps.

4 Let n1 < n2 < . . . be a sequence of natural numbers such that for i < j the decimal representation of ni does not occur as the leftmost digits of the decimal representation
P 1ofnj
. (For example, 137and 13729 cannot both occur in the sequence.) Prove that:
i ni
1
1
1
1
1
1
1
1 + 2 + 3 + 4 + 5 + 6 + 7 + + 9 ! s:D img src=SMILIESP AT H/iconm rgreen.gif alt =
: Dtitle = M r.Green/ ><! s : D ><! s : D >< imgsrc =
SM ILIESP AT H/iconm rgreen.gif alt = : Dtitle = M r.Green/ ><! s : D >
LetABCbeatriangleandDbethepointontheextensionof sideBCpastCsuchthatCD = AC.T hecircumcircleof AC
Farecollinear.
56 Let K be a convex polygon in the plane. Show that for any triangle of the minimum possible
area containing K, the midpoints of its sides lie on K

This file was downloaded from the AoPS Math Olympiad Resources Page
http://www.artofproblemsolving.com/

Page 3

iran
National Math Olympiad (3rd Round)
1999

1 Define the sequence (xn ) by x0 = 1 and for all n N,



xn1 + (3r 1)/2, if n = 3r1 (3k + 1);
xn =
xn1 (3r + 1)/2, if n = 3r1 (3k + 2).
where k N0 , r N. Prove that every integer occurs in this sequence exactly once.
2 Let n(r) be the maximum possible number of points
with integer coordinates on a circle with

3
radius r in Cartesian plane. Prove that n(r) < 6 3r2 .
3 Let ABCDEF be a convex hexagon such that AB = BC, CD = DE and EF = F A. Prove
that
CD EF
3
AB
+
+
.
AD
CF
EB
2
4 Find all functions f : R R such that for all x, y,
f (f (x) + y) = f (x2 y) + 4f (x)y.
5 In a triangle ABC, the bisector of angle BAC intersects BC at D. The circle through A
which is tangent to BC at D meets AC again at M . Line BM meets again at P . Prove
that line AP is a median of 4ABD.
6 Let ABC be a given triangle. Consider any painting of points of the plane in red and green.
Show that there exist either two red points on the distance 1, or three green points forming
a triangle congruent to 4ABC.

This file was downloaded from the AoPS Math Olympiad Resources Page
http://www.artofproblemsolving.com/

Page 1

iran
National Math Olympiad (3rd Round)
2001

1 Find all functions f : Q Q such that: f (x) + f ( x1 ) = 1 2f (f (x)) = f (2x)


2 Does there exist a sequence {bi }
i=1 of positive real numbers such that for each natural m:
bm + b2m + b3m + =

1
m

This file was downloaded from the AoPS Math Olympiad Resources Page
http://www.artofproblemsolving.com/

Page 1

iran
National Math Olympiad (3rd Round)
2002

1 Let a, b, c Rn , a + b + c = 0 and > 0. Prove that


Y |a| + |b| + (2 + 1)|c|
(2 + 3)3
|a| + |b| + |c|

cycle

2 f : R R+ is a non-decreasing function. Prove that there is a point a R that


f (a +

1
) < 2f (a)
f (a)

3 an is a sequence that a1 = 1, a2 = 2, a3 = 3, and


an+1 = an an1 +

a2n
an2

Prove that for each natural n, an is integer.


4 an (n is integer) is a sequence from positive reals that
an

an+2 + an+1 + an1 + an2


4

Prove an is constant.
5 is circumcirlce of triangle ABC. We draw a line parallel to BC that intersects AB, AC at
E, F and intersects at U, V . Assume that M is midpoint of BC. Let 0 be circumcircle of
U M V . We know that R(ABC) = R(U M V ). M E and 0 intersect at T , and F T intersects
0 at S. Prove that EF is tangent to circumcircle of M CS.

6 M is midpoint of BC.P is an arbitary point on BC. C1 is tangent to big circle.Suppose radius of C1


is r1 Radius of C4 is equal to radius of C1 and C4 is tangent to BC at P. C2 and C3 are tangent to big
circle and line BC and circle C4 . [img]http://aycu01.webshots.com/image/4120/2005120338156776027r s.jpg[/img]
r1 + r2 + r3 = R(R radius of big circle)
In triangle ABC, AD is angle bisector (D is on BC) if AB + AD = CD and AC + AD = BC,
what are the angles of ABC?
Circles C1 and C2 are tangent to each other at K and are tangent to circle C at M and N . External
tangent of C1 and C2 intersect C at A and B. AK and BK intersect with circle C at E and F
respectively. If AB is diameter of C, prove that EF and M N and OK are concurrent. (O is center
of circle C.)
Let M and N be points on the side BC of triangle ABC, with the point M lying on the segment
BN , such that BM = CN . Let P and Q be points on the segments AN and AM , respectively,
such that ]P M C = ]M AB and ]QN B = ]N AC. Prove that ]QBC = ]P CB.

This file was downloaded from the AoPS Math Olympiad Resources Page
http://www.artofproblemsolving.com/

Page 1

iran
National Math Olympiad (3rd Round)
2002

H, I, O, N are orthogonal center, incenter, circumcenter, and Nagelian point of triangle ABC.
Ia , Ib , Ic are excenters of ABC corresponding vertices A, B, C. S is point that O is midpoint of
HS. Prove that centroid of triangles Ia Ib Ic and SIN concide.
In an m n table there is a policeman in cell (1, 1), and there is a thief in cell (i, j). A move is
going from a cell to a neighbor (each cell has at most four neighbors). Thief makes the first move,
then the policeman moves and ... For which (i, j) the policeman can catch the thief?
We have a bipartite graph G (with parts X and Y ). We orient each edge arbitrarily. Hessam chooses
a vertex at each turn and reverse the orientation of all edges that v is one of their endpoint. Prove
that with these steps we can reach to a graph that for each vertex v in part X, deg + (v) deg (v)
and for each vertex in part Y , deg + v deg v
f, g are two permutations of set X = {1, . . . , n}. We say f, g have common points iff there is a
k X that f (k) = g(k). a) If m > n2 , prove that there are m permutations f1 , f2 , . . . , fm from X
that for each permutation f X, there is an index i that f, fi have common points. b) Prove that
if m n2 , we can not find permutations f1 , f2 , . . . , fm satisfying the above condition.
A subset S of N is eventually linear iff there are k, N N that for n > N, n S k|n. Let S
be a subset of N that is closed under addition. Prove that S is eventually linear.
Let A be be a point outside the circle C, and AB and AC be the two tangents from A to this
circle C. Let L be an arbitrary tangent to C that cuts AB and AC in P and Q. A line through
P parallel to AC cuts BC in R. Prove that while L varies, QR passes through a fixed point.
! s:) img src=SMILIESP AT H/smile.gif alt = :)title = Smile/ ><! s :) >
F orpositivea,b,c, a2 + b2 + c2 + abc = 4Prove a + b + c 3
Find the smallest natural number n that the following statement holds : Let A be a finite subset
of R2 . For each n points in A there are two lines including these n points. All of the points lie on
two lines.
Find all continious f : R R that for any x, y
f (x) + f (y) + f (xy) = f (x + y + xy)
I is incenter of triangle ABC. Incircle of ABC touches AB, AC at X, Y . XI intersects incircle at
M . Let CM AB = X 0 . L is a point on the segment X 0 C that X 0 L = CM . Prove that A, L, I
are collinear iff AB = AC.
a0 = 2, a1 = 1 and for n 1 we know that : an+1 = an + an1 m is an even number and p is prime
number such that p divides am 2. Prove that p divides am+1 1.
Excircle of triangle ABC corresponding vertex A, is tangent to BC at P . AP intersects circumcircle
of ABC at D. Prove
r(P CD) = r(P BD)
whcih r(P CD) and r(P BD) are inradii of triangles P CD and P BD.

This file was downloaded from the AoPS Math Olympiad Resources Page
http://www.artofproblemsolving.com/

Page 2

iran
National Math Olympiad (3rd Round)
2002

15000 years ago Tilif ministry in Persia decided to define a code for n 2 cities. Each code is
a sequence of 0, 1 such that no code start with another code. We knowP
that from 2m calls from
foreign countries to Persia 2mai of them where from the i-th city (So ni=1 21ai = 1). Let li be
P
length of code assigned to i-th city. Prove that ni=1 2lii is minimum iff i, li = ai
Find all polynomials p with real coefficients that if for a real a,p(a) is integer then a is integer.
A, B, C are on circle C. I is incenter of ABC , D is midpoint of arc BAC. W is a circle that is
tangent to AB and AC and tangent to C at P . (W is in C) Prove that P and I and D are on a
line.
An ant walks on the interior surface of a cube, he moves on a straight line. If ant reaches to an edge
the he moves on a straight line on cubes net. Also if he reaches to a vertex he will return his path.
a) Prove that for each beginning point ant can has infinitely many choices for his direction that
its path becomes periodic. b) Prove that if if the ant starts from point A and its path is periodic,
then for each point B if ant starts with this direction, then his path becomes periodic.

This file was downloaded from the AoPS Math Olympiad Resources Page
http://www.artofproblemsolving.com/

Page 3

iran
National Math Olympiad (3rd Round)
2003

1 suppose this equation: x sup2/sup +y sup2/sup +z sup2/sup =w sup2/sup .


show that the solution of this equation ( if w,z have same parity) are in this form: x=2d(XZYW), y=2d(XW+YZ),z=d(X sup2/sup +Y sup2/sup -Z sup2/sup -W sup2/sup
),w=d(X sup2/sup +Y sup2/sup +Z sup2/sup +W sup2/sup )
2 assume ABCD a convex quadrilatral. P and Q are on BC and DC respectively such that
angle BAP= angle DAQ .prove that [ADQ]=[ABP] ([ABC] means its area ) iff the line which
crosses through the orthocenters of these traingles , is perpendicular to AC.

3 assume that A is finite subset of prime number and a is an positive integer prove that ther are
finite positive integer like m s.t: prime divisors of am 1arecontainedinA.XOY isangleintheplane.A, Barevar
1/OB = 1/K(kisconstant).drawtwocircleswithdiameterOAandOB.provethatcommonexternaltangenttoth
45 assume P is a odd prime number and S is the sum of the all primitive root mod P. show that If p-1
isnt empty of perfect square( I mean that we can show p-1 =k sup2/supm) then S=0(mod P).
if not S=?(mod p).
6 let the incircle of a triangle ABC touch BC,AC,AB at A1,B1,C1 respectively. M and N are the
midpoints of AB1 and AC1 respectively. MN meets A1C1 at T . draw two tangents TP and TQ
through T to incircle. PQ meets MN at L and B1C1 meets PQ at K . assume I is the center of the
incircle .
prove IK is parallel to AL
7 f1 , f2 , . . . , fn are polynomials with integer coefficients. Prove there exist a reducible g(x) with
integer coefficients that f1 + g, f2 + g, . . . , fn + g are irreducible.
8 Lets call perfect power any positive integer n such that n = ab for some integers a and b, with
b > 1. a) Find 2004 perfect powers in arithmetic progression. b) Prove that perfect powers cannot
form an infinite arithmetic progression.
9 Does there exist an infinite set S such that for every a, b S we have a2 + b2 ab | (ab)2 .

10 let p be a prime and a and n be natural numbers such that (pa 1)/(p1) = 2n f indthenumberof naturaldivisorsof n
! s :) >< imgsrc = SM ILIESP AT H/smile.gif alt = :)title = Smile/ ><! s :
) > assumethatXisasetof nnumber.and0 k n.the maximum number of permutation which
acting on X st every two of them have at least k component in common,is an,k .and the maximum
nuber of permutation st every two of them have at most k component in common,is bn,k . a)proeve
that :an,k bn,k1 n! b)assume that p is prime number,determine the exact value of ap,2 .
12 There is a lamp in space.(Consider lamp a point) Do there exist finite number of equal sphers in
11
space that the light of the lamp can not go to the infinite?(If a ray crash in a sphere it stops)

This file was downloaded from the AoPS Math Olympiad Resources Page
http://www.artofproblemsolving.com/

Page 1

iran
National Math Olympiad (3rd Round)
2003

13 here is the most difficult and the most beautiful problem occurs in 21th iranian (2003) olympiad
assume that P is n-gon ,lying on the plane ,we name its edge 1,2,..,n. if S=s1,s2,s3,.... be a finite
or infinite sequence such that for each i, si is in 1,2,...,n, we move P on the plane according to
the S in this form: at first we reflect P through the s1 ( s1 means the edge which iys number is
s1)then through s2 and so on like the figure below. a)show that there exist the infinite sequence S
sucth that if we move P according to S we cover all the plane b)prove that the sequence in a) isnt
periodic. c)assume that P is regular pentagon ,which the radius of its circumcircle is 1,and D is
circle ,with radius 1.00001 ,arbitrarily in the plane .does exist a sequence S such that we move P
according to S then P reside in D completely?

14 n 6isaninteger.evaluatetheminimumof f (n)s.t : anygraphwithnverticesandf (n)edgecontainstwocyclewhichare


matrix which is filled with just 0, 1 and any two row differ in at least n/2 members, show that
m 2n. ( for example the diffrence of this two row is only in one index 110 100)
Edited by Myth
16 Segment AB is fixed in plane. Find the largest n, such that there are n points P1 , P2 , . . . , Pn in
15
plane that triangles ABPi are similar for 1 i n. Prove that all of Pi s lie on a circle.

17 A simple calculator is given to you. (It contains 8 digits and only does the operations +,-,*,/, )

How can you find 3

with accuracy of 6 digits.

18 In tetrahedron ABCD, radius four circumcircles of four faces are equal. Prove that AB = CD,
AC = BD and AD = BC.
19 An integer n is called a good number if and only if |n| is not square of another intger. Find all
integers m such that they can be written in infinitely many ways as sum of three different good
numbers and product of these three numbers is square of an odd number.
20 Suppose that M is an arbitrary point on side BC of triangle ABC. B1 , C1 are points on AB, AC
such that M B = M B1 and M C = M C1 . Suppose that H, I are orthocenter of triangle ABC and
incenter of triangle M B1 C1 . Prove that A, B1 , H, I, C1 lie on a circle.
21 Let ABC be a triangle. Wa is a circle with center on BC passing through A and perpendicular to
circumcircle of ABC. Wb , Wc are defined similarly. Prove that center of Wa , Wb , Wc are collinear.
22 Let a1 = a2 = 1 and
n(n + 1)an+1 + n2 an + 5
2
n+2
for each n N. Find all n such that an N.
an+2 =

23 Find all homogeneous linear recursive sequences such that there is a T such that an = an+T for
each n.

This file was downloaded from the AoPS Math Olympiad Resources Page
http://www.artofproblemsolving.com/

Page 2

iran
National Math Olympiad (3rd Round)
2003

24 A, B are fixed points. Variable line l passes through the fixed point C. There are two circles passing
through A, B and tangent to l at M, N . Prove that circumcircle of AM N passes through a fixed
point.
25 Let A, B, C, Q be fixed points on plane. M, N, P are intersection points of AQ, BQ, CQ with
BC, CA, AB. D0 , E 0 , F 0 are tangency points of incircle of ABC with BC, CA, AB. Tangents
drawn from M, N, P (not triangle sides) to incircle of ABC make triangle DEF . Prove that
DD0 , EE 0 , F F 0 intersect at Q.
26 Circles C1 , C2 intersect at P . A line is drawn arbitrarily from P and intersects with C1 , C2 at
B, C. What is locus of A such that the median of AM of triangle ABC has fixed length k.
27 S N is called a square set, iff for each x, y S, xy + 1 is square of an integer. a) Is S finite? b)
Find maximum number of elements of S.
28 There are n points in R3 such that every three form an acute angled triangle. Find maximum of n.
29 Let c C and Ac = {p C[z]|p(z 2 + c) = p(z)2 + c}.
a) Prove that for each c C, Ac is infinite.
b) Prove that if p A1 , and p(z0 ) = 0, then |z0 | < 1.7.
c) Prove that each element of Ac is odd or even.
Let fc = z 2 + c C[z]. We see easily that Bc := {z, fc (z), fc (fc (z)), . . . } is a subset of Ac . Prove
that in the following cases Ac = Bc .
d) |c| > 2.
e) c Q\Z.
f) c is a non-algebraic number
g) c is a real number and c 6 [2, 14 ].

This file was downloaded from the AoPS Math Olympiad Resources Page
http://www.artofproblemsolving.com/

Page 3

iran
National Math Olympiad (3rd Round)
2004

1 We say m n for natural m,n nth number of binary representation of m is 1 or mth


number of binary representation of n is 1. and we say m n if and only if m, n doesnt have
the relation We say A N is golden U, V A that are finite and arenot empty and
U V = ,There exist z A that x U, y V we have z x, z y Suppose P is set of prime
numbers.Prove if P = P1 ... Pk and Pi Pj = then one of P1 , ..., Pk is golden.
2 A is a convex set in plane prove taht ther exist O in A that for every line XX 0 passing throw
O and X and X 0 are boundry points of A then
1
OX

2
2
OX 0
3 Suppose V = Zn2 and for a vector x = (x1 , ..xn ) in V and permutation .We have x =
(x(1) , ..., x(n) ) Suppose n = 4k + 2, 4k + 3 and f : V 7 V is injective and if x and y differ
in more than n/2 places then f (x) and f (y) differ in more than n/2 places. Prove there exist
permutaion and vector v that f (x) = x + v
4 We have finite white and finite black points that for each 4 oints there is a line that white
points and black points are at different sides of this line.Prove there is a line that all white
points and black points are at different side of this line.
5 assume that k,n are two positive integer k ncount the number of permutation { 1, . . . , n} st
for any 1 i, j kand any positive integer m we have f m (i) 6= j (f m meas iterarte function,)
6 assume that we have a n*n table we fill it with 1,...,n such that each number exists exactly

n times prove that there exist a row or column such that at least n diffrent number are
contained.
7 Suppose F is a polygon with lattice vertices and sides parralell to x-axis and y-axis.Suppose
S(F ), P (F ) are area and perimeter of F . Find the smallest k that: S(F ) k.P (F )2
8 P is a n-gon with sides l1 , ..., ln and vertices on a circle. Prove that no n-gon with this sides
has area more than P
9 Let ABC be a triangle, and O the center of its circumcircle. Let a line through the point O
intersect the lines AB and AC at the points M and N , respectively. Denote by S and R the
midpoints of the segments BN and CM , respectively. Prove that ]ROS = ]BAC.
10 f : R2 7 R2 is injective and surjective.Distance of X and Y is not less than distance of f (X)
and f (Y ).Prove for A in plane:
S(A) S(f (A))
Which S(A) is area of A

This file was downloaded from the AoPS Math Olympiad Resources Page
http://www.artofproblemsolving.com/

Page 1

iran
National Math Olympiad (3rd Round)
2004

11 assume that ABC is acute traingle and AA is median we extend it until it meets circumcircle
at A. let APa be a diameter of the circumcircle. the pependicular from A to APa meets
the tangent to circumcircle at A in the point Xa ; we define Xb , Xc similary . prove that
Xa , Xb , Xc are one a line.
12 N10 is generalization of N that every hypernumber in N10 is something like: ...a2 a1 a0 with
ai 0, 1..9 (Notice that ...000 N10 ) Also we easily have +, in N10 . first k number of a b=
first k nubmer of (first k number of a * first k number of b) first k number of a + b= first k
nubmer of (first k number of a + first k number of b) Fore example ...999 + ...0001 = ...000
Prove that every monic polynomial in N10 [x] with degree d has at most d2 roots.
13 Suppose f is a polynomial in Z[X] and m is integer .Consider the sequence ai like this a1 = m
and ai+1 = f (ai ) find all polynomials f and alll integers m that for each i:
ai |ai+1
14 We define f : N N, f (n) =

Pn

k=1 (k, n).

a) Show that if gcd(m, n) = 1 then we have f (mn) = f (m) f (n);


P
b) Show that d|n f (d) = nd(n).
15 This problem is easy but nobody solved it. point A moves in a line with speed v and B
moves also with speed v 0 that at every time the direction of move of B goes from A.We know
v v 0 .If we know the point of beginning of path of A, then B must be where at first that B
can catch A.
16 Let ABC be a triangle . Let point X be in the triangle and AX intersects BC in Y . Draw the
perpendiculars Y P, Y Q, Y R, Y S to lines CA, CX, BX, BA respectively. Find the necessary
and sufficient condition for X such that P QRS be cyclic .
17 Let p = 4k + 1 be a prime. Prove that p has at least

(p1)
2

primitive roots.

18 Prove that P
for each n, there is a subset of N such as {a1 , . . . , an } that for each subset S of
{1, . . . , n}, iS ai has the same set of prime divisors.
19 Find all integer solutions of p3 = p2 + q 2 + r2 where p, q, r are primes.
20 p(x) is a polynomial in Z[x] such that for each m, n N there is an integer a such that
n|p(am ). Prove that 0 or 1 is a root of p(x).
P
21 a1 , a2 , . . . , an are integers and i,j (ai aj )2 > 0. Prove that there are infinitely many prime
numbers such as p that for some k:
p|ak1 + + akn

This file was downloaded from the AoPS Math Olympiad Resources Page
http://www.artofproblemsolving.com/

Page 2

iran
National Math Olympiad (3rd Round)
2004

22 Suppose that F is a family of subsets of X. A, B are two subsets of X s.t. each element of
X has non-empty intersection with A, B. We know that no subset of X with n 1 elements
has this property. Prove that there is a representation A, B in the form A = {a1 , . . . , an } and
B = {b1 , . . . , bn } such that for each 1 i n, there is an element of F containing both ai , bi .
23 F is a family of 3-subsets of set X. Every two distinct elements of X are exactly in k elements
of F. It is known that there is a partition of F to sets X1 , X2 such that each element of F
has non-empty intersection with both X1 , X2 . Prove that |X| 4.
24 In triangle ABC, points M, N lie on line AC such that M A = AB and N B = N C. Also
K, L lie on line BC such that KA = KB and LA = LC. It is know that KL = 21 BC and
M N = AC. Find angles of triangle ABC.
25 Finitely many convex subsets of R3 is given such that every three have non-empty intersection.
Prove that there is a line in R3 such that intersects all of these subsets.
26 Finitely many points are given on the surface of a sphere, such that every four of them lie on
the surface of open hemisphere. Prove that all points lie on the surface of an open hemisphere.
27 1 , . . . , n are n concurrent segments (their lines concur) in the real plane. Prove that if for
every three of them there is a line intersecting these three segments, then there is a line that
interesects all of segments.
28 Find all prime numbers such that p = m2 + n2 and p|m3 + n3 4.
29 Incircle of triangle ABC touches AB, AC at P, Q. BI, CI intersect with P Q at K, L. Prove
that circumcircle of ILK is tangent to incircle of ABC if and only if AB + AC = 3BC.
30 Find all polynomials p Z[x] such that (m, n) = 1 (p(m), p(n)) = 1

This file was downloaded from the AoPS Math Olympiad Resources Page
http://www.artofproblemsolving.com/

Page 3

iran
National Math Olympiad (3rd Round)
2005

Day 1
1 Suppose a, b, c R+ . Prove that :
1 1 1
a b c
( + + )2 (a + b + c)( + + )
b c a
a b
c
2 Suppose {xn } is a decreasing sequence that lim xn = 0. Prove that
n

P
(1)n xn is convergent
n

3 Find all > 0 and > 0 that for each (x1 , . . . , xn ) and (y1 , . . . , yn ) R+ that:
X
X
X
(
xi )(
yi )
xi yi
4 Suppose P, Q R[x] that deg P = deg Q and P Q0 QP 0 has no real root. Prove that for
each R number of real roots of P and P + (1 )Q are equal.
5 Suppose a, b, c R+ and
a2
Prove that ab + ac + bc

1
1
1
+ 2
+ 2
=2
+1 b +1 c +1

3
2

6 Suppose A Rm is closed and non-empty . f : A 7 A is a lipchitz function with constant


less than 1. (ie there exist c < 1 that |f (x) f (y)| < |x y|, x, y A). Prove that f has a
unique point like x that f (x) = x

This file was downloaded from the AoPS Math Olympiad Resources Page
http://www.artofproblemsolving.com/

Page 1

iran
National Math Olympiad (3rd Round)
2005

Day 2
1 From each vertex of triangle ABC we draw 3 arbitary parrallell lines, and from each vertex
we draw a perpendicular to these lines. There are 3 rectangles that one of their diagnals is
triangles side. We draw their other diagnals and call them `1 , `2 and `3 .
a) Prove that `1 , `2 and `3 are concurrent at a point P .
b) Find the locus of P as we move the 3 arbitary lines.
2 Suppose O is circumcenter of triangle ABC. Suppose S(OAB)+S(OAC)
= S(OBC). Prove
2
that ditsance of O (circumcenter) from radial axis of circumcircle and 9-point circle is
a2
p

9R2 (a2 + b2 + c2 )

3 Prove that in acute-angled traingle ABC if r is inradius and R is radius of circumcircle then:
a2 + b2 + c2 4(R + r)2
4 Suppose in triangle ABC incircle touches the side BC at P and AP B = . Prove that :
1
2
1
+
=
pb pc
rtg
5 Suppose H and O are orthocenter and circumcenter of triangle ABC. is circumcircle of
ABC. AO intersects with at A1 . A1 H intersects with at A0 and A00 is the intersection
point of and AH. We define points B 0 , B 00 , C 0 and C 00 similiarly. Prove that A0 A00 , B 0 B 00
and C 0 C 00 are concurrent in a point on the Euler line of triangle ABC.

This file was downloaded from the AoPS Math Olympiad Resources Page
http://www.artofproblemsolving.com/

Page 2

iran
National Math Olympiad (3rd Round)
2005

Day 3
1 Find all n, p, q N that:

2n + n2 = 3p 7q

2 a N and m = a2 + a + 1. Find the teh number of 0 x m that:


x3 1(mod m)
3 p(x) is an irreducible polynomial in Q[x] that deg p is odd. q(x), r(x) are polynomials with
rational coefficients that p(x)|q(x)2 + q(x).r(x) + r(x)2 . Prove that
p(x)2 |q(x)2 + q(x).r(x) + r(x)2
4 k is an integer. We define the sequence {an }
n=0 like this:
a0 = 0,

a1 = 1,

an = 2kan1 (k 2 + 1)an2 (n 2)

p is a prime number that p 3(mod 4) a) Prove that an+p2 1 an (mod p) b) Prove that
an+p3 p an (mod p2 )
5 a, b, c N that a, b 6= c. Prove that there are infinitely many prime numbers like P that
there exist n N that p|an + bn cn

This file was downloaded from the AoPS Math Olympiad Resources Page
http://www.artofproblemsolving.com/

Page 3

iran
National Math Olympiad (3rd Round)
2005

Day 4
1 We call the set A Rn CN if and only if every continuous f : A 7 A ther exist : x A that
f (x) = x a)Example: We know that A = {x Rn ||x| 1} is CN b) circle is not CN. Which
one of thes sets are CN? 1) A = {x R3 ||x| = 1}
2) The cross {(x, y) R2 |xy = 0, |x| + |y| 1}
3) Graph of f : [0, 1] :7 R:
f (x) = sin

1
if x 6= 0, f (0) = 0
x

2 n vectors are on the plane. We can move each vector forward and backeard on the line that
the vector is on it. If there are 2 vectors that their endpoints concide we can omit them and
replace them with their sum (If their sum is nonzero). Suppose with these operations with 2
different method we reach to a vector. Prove that these vectors are on a common line
3 f (n) is the least number that there exist a f (n)mino that contains every nmino. Prove
that 10000 f (1384) 960000. Find some bound for f (n)
4 a) Year 1872 Texas 3 gold miners found a peice of gold. They have a coin that with possibility
of 12 it will come each side, and they want to give the piece of gold to one of themselves
depending on how the coin will come. Design a fair method (It means that each of the 3
miners will win the piece of gold with possibility of 13 ) for the miners.
b) Year 2005, faculty of Mathematics, Sharif university of Technolgy Suppose 0 < < 1 and
we want to find a way for people name A and B that the possibity of winning of A is . Is
it possible to find this way?
c) Year 2005 Ahvaz, Takhti Stadium Two soccer teams have a contest. And we want to
choose each players side with the coin, But we dont know that our coin is fair or not. Find
a way to find that coin is fair or not?
d) Year 2005,summer In the National mathematical Oympiad in Iran. Each student has a
coin and must find a way that the possibility of coin being TAIL is or no. Find a way for
the student.

This file was downloaded from the AoPS Math Olympiad Resources Page
http://www.artofproblemsolving.com/

Page 4

iran
National Math Olympiad (3rd Round)
2005

Day 5
1 An airplane wants to go from a point on the equator, and at each moment it will go to the
northeast with speed v. Suppose the radius of earth is R. a) Will the airplane reach to the
north pole? If yes how long it will take to reach the north pole? b) Will the airplne rotate
finitely many times around the north pole? If yes how many times?
2 We define a relation between subsets of Rn . A B we can partition A, B in sets
n
n
[
[
A1 , . . . , An and B1 , . . . , Bn (i.e A =
Ai , B =
Bi , Ai Aj = , Bi Bj = ) and
i=1

i=1

Ai ' Bi . Say the the following sets have the relation or not ?
a) Natural numbers and composite numbers.
b) Rational numbers
and rational numbers with

finite digits in base 10. c) {x Q|x < 2} and {x Q|x < 3} d) A = {(x, y) R2 |x2 +y 2 <
1} and A \ {(0, 0)}
Q
Q
3 For each m N we define rad (m) = pi that m = pi i .
abc Conjecture Suppose  > 0 is an arbitary number, then there exist K depinding on 
that for each 3 numbers a, b, c Z that gcd(a, b) = 1 and a + b = c then:
max{|a|, |b|, |c|} K(rad (abc))1+
Now prove each of the foollowing statements with abc conjectures
: a) Fermats last theorem
Q i
for n > N that N is a natural number. b)We call n = pi strong if and only i 2 c)
Prove that there are finitely many n that n, n + 1, n + 2 are strong. d) Prove that there are
finitely many rational numbers like pq that:

p 21384
3
2 < 3
q
q
4 Suppose we have some proteins that each protein is a sequence of 7 AMINO-ACIDS
A, B, C, H, F, N . For example AF HN N N HAF F C is a protein. There are some steps that
in each step an amino-acid will change to another one. For example with the step N A N
the protein BAN AN A will cahnge to BAN N A(in Persian means workman). We have a
set of allowed steps that each protein can change with these steps. For example with the set
of steps:
1) AA A
2) AB BA
3) A null Protein ABBAABA will change like this:
ABBAABA

This file was downloaded from the AoPS Math Olympiad Resources Page
http://www.artofproblemsolving.com/

Page 5

iran
National Math Olympiad (3rd Round)
2005

ABBABA
BABABA
BBAABA
BBABA
BBBAA
BBBA
BBB You see after finite steps this protein will finish it steps. Set of allowed steps that
for them there exist a protein that may have infinitely many steps
 is dangerous. Which of
HHCC HCCH
the following allowed sets are dangerous? a) N O OON N b)
c)
CC CH
. . . B} d) Design a set of allowed
Design a set of allowed steps that change AA
. . . A} BB
| {z
| {z
n

2n

. . B} CC
. . . C} You see from c and d that we acn calculate
steps that change A
. . A} B
| .{z
| {z
| .{z
n

mn

the functions F (n) = 2n and G(M, N ) = mn with these steps. Find some other calculatable
functions with these steps. (It has some extra mark.)

This file was downloaded from the AoPS Math Olympiad Resources Page
http://www.artofproblemsolving.com/

Page 6

iran
National Math Olympiad (3rd Round)
2006

Algebra

1 For positive numbers x1 , x2 , . . . , xs , we know that


s
X
k=1

xm
k

Qs

s
X

i=1 xk

= 1. Prove that for each m n

xnk

k=1

2 Find all real polynomials that


p(x + p(x)) = p(x) + p(p(x))
3 Find all real x, y, z that

x + y + zx =

y + z + xy =

z + x + yz =

1
2
1
2
1
2

4 p(x) is a real polynomial that for each x 0, p(x) 0. Prove that there are real polynomials
A(x), B(x) that p(x) = A(x)2 + xB(x)2
5 Find the biggest real number k such that for each right-angled triangle with sides a, b, c, we
have
a3 + b3 + c3 k (a + b + c)3 .
6 P, Q, R are non-zero polynomials that for each z C, P (z)Q(
z ) = R(z). a) If P, Q, R R[x]
prove that Q is constant polynomial. b) IS the above statement correct for P, Q, R C[x]?

This file was downloaded from the AoPS Math Olympiad Resources Page
http://www.artofproblemsolving.com/

Page 1

iran
National Math Olympiad (3rd Round)
2006

Combinatorics

1 Let A be a family of subsets of {1, 2, . . . , n}


 such that no member of A is contained in another.
n
Sperners Theorem states that |A| b n c . Find all the families for which the equality holds.
2

2 Let B be a subset of Zn3 with the property that for every two distinct members (a1 , . . . , an ) and
(b1 , . . . , bn ) of B there exist 1 i n such that ai bi + 1 (mod 3). Prove that |B| 2n .
3 Let C be a (probably infinite) family of subsets of N such that for every chain C1 C2 . . .
of members of C, there is a member of C containing all of them. Show that there is a member
of C such that no other member of C contains it!
4 Let D be a family of s-element subsets of {1. . . . , n} such that every k members of D have
non-empty intersection. Denote by D(n, s, k) the maximum cardinality of such a family. a)
Find D(n, s, 4). b) Find D(n, s, 3).
5 Let E be a family of subsets of {1, 2, . . . , n} with the property that for each A {1, 2, . . . , n}
n+d
there exist B F such that nd
2 |A 4 B| 2 . (where A 4 B = (A \ B) (B \ A) is the
symmetric difference). Denote by f (n, d) the minimum cardinality of such a family. a) Prove
n
that if n is even then f (n, 0) n. b) Prove that if n d is even then f (n, d) d d+1
e. c)
Prove that if n is even then f (n, 0) = n
6 The National Foundation of Happiness (NFoH) wants to estimate the happiness of people of
country. NFoH selected n random persons, and on every morning asked from each of them
whether she is happy or not. On any two distinct days, exactly half of the persons gave the
same answer. Show that after k days, there were at most n nk persons whose yes answers
equals their no answers.

This file was downloaded from the AoPS Math Olympiad Resources Page
http://www.artofproblemsolving.com/

Page 2

iran
National Math Olympiad (3rd Round)
2006

Final Exam

1 A regular polyhedron is a polyhedron that is convex and all of its faces are regular polygons.
We call a regular polhedron a Choombam iff none of its faces are triangles. a) prove that each
choombam can be inscribed in a sphere. b) Prove that faces of each choombam are polygons of
at most 3 kinds. (i.e. there is a set {m, n, q} that each face of a choombam is n-gon or m-gon
or q-gon.) c) Prove that there is only one choombam that its faces are pentagon and hexagon.
(Soccer ball) [img]http://aycu08.webshots.com/image/5367/2001362702285797426r s.jpg[/img]d)F orn3, apr
23 For A Z and a, b Z. We define aA + b := {ax + b|x A}. If a 6= 0 then we calll aA + b and
A to similar sets. In this question the Cantor set C is the number of non-negative integers that in
their base-3 representation there is no 1 digit. You see

C = (3C)(3C
+ 2)

(1)

(i.e. C is partitioned to sets 3C and 3C +2). We give another example C = (3C)(9C


+6)(3C
+2).
A representation of C is a partition of C to some similiar sets. i.e.
C=

n
[

Ci

(2)

i=1

and Ci = ai C + bi are similar to C. We call a representation of C a primitive representation iff


union of some of Ci is not a set similar and not equal to C. Consider a primitive representation
of Cantor set. Prove that a) ai > 1. b) ai are powers of 3. c) ai > bi d) (1) is the only primitive
representation of C.

4 The image shown below is a cross with length 2. If length of a cross of length k it is called a k-cross.
(Each k-cross ahs 6k+1 squares.) [img]http://aycu08.webshots.com/image/4127/2003057947601864020t h.jpg[/img
5 space can not be tiled with k-crosses.

5 A calculating ruler is a ruler for doing algebric calculations. This ruler has three arms, two of them
are sationary and one can move freely right and left. Each of arms is gradient. Gradation of each
arm depends on the algebric operation ruler does. For eaxample the ruler below is designed for multiplying two numbers. Gradations are logarithmic. [img]http://aycu05.webshots.com/image/5604/20004685171623
Grade first arm (x) and (y) from 1 to 10. 2) Find all rulers that do the multiplication in the interval
[1, 10]. 3) Prove that there is not a ruler for calculating x2 + xy + y 2 , that its first and second arm
are grade from 0 to 10.
6 Assume that C is a convex subset of Rd . Suppose that C1 , C2 , . . . , Cn are translations of C that
Ci C 6= but Ci Cj = . Prove that
n 3d 1

This file was downloaded from the AoPS Math Olympiad Resources Page
http://www.artofproblemsolving.com/

Page 3

iran
National Math Olympiad (3rd Round)
2006

Prove that 3d 1 is the best bound. P.S. In the exam problem was given for n = 3.

7 We have finite number of distinct shapes in plane. A convex Kearting of these shapes is covering
plane with convex sets, that each set consists exactly one of the shapes, and sets intersect at most in
border. [img]http://aycu30.webshots.com/image/4109/2003791140004582959t h.jpg[/img]InwhichcaseConvexkea
3 points that are not collinear in Qn
a) Suppose that ABC is triangle in Qn . Prove that there is a triangle A0 B 0 C 0 in Q5 that B 0 A0 C 0 =
BAC. b) Find a natural m that for each traingle that can be embedded in Qn it can be embedded
in Qm . c) Find a triangle that can be embedded in Qn and no triangle similar to it can be embedded
in Q3 . d) Find a natural m0 that for each traingle that can be embedded in Qn then there is a
triangle similar to it, that can be embedded in Qm . You must prove the problem for m = 9 and
m0 = 6 to get complete mark. (Better results leads to additional mark.)

This file was downloaded from the AoPS Math Olympiad Resources Page
http://www.artofproblemsolving.com/

Page 4

iran
National Math Olympiad (3rd Round)
2006

Geometry

1 Prove that in triangle ABC, radical center of its excircles lies on line GI, which G is Centroid
of triangle ABC, and I is the incenter.
2 ABC is a triangle and R, Q, P are midpoints of AB, AC, BC. Line AP intersects RQ in E
and circumcircle of ABC in F . T, S are on RP, P Q such that ES P Q, ET RP . F 0
is on circumcircle of ABC that F F 0 is diameter. The point of intersection of AF 0 and BC
is E 0 . S 0 , T 0 are on AB, AC that E 0 S 0 AB, E 0 T 0 AC. Prove that T S and T 0 S 0 are
perpendicular.
3 In triangle ABC, if L, M, N are midpoints of AB, AC, BC. And H is orthogonal center of
triangle ABC, then prove that
1
LH 2 + M H 2 + N H 2 (AB 2 + AC 2 + BC 2 )
4
4 Circle (O, R) and its chord AB is given. Suppose C is midpoint of arc AB. X is an arbitrary
point on the cirlce. Perpendicular from B to CX intersects circle again in D. Perpendicular
from C to DX intersects circle again in E. We draw three lines `1 , `2 , `3 from A, B, E parralell
to OX, OD, OC. Prove that these lines are concurrent and find locus of concurrncy point.
5 M is midpoint of side BC of triangle ABC, and I is incenter of triangle ABC, and T is
midpoint of arc BC, that does not contain A. Prove that
cos B + cos C = 1 M I = M T

This file was downloaded from the AoPS Math Olympiad Resources Page
http://www.artofproblemsolving.com/

Page 5

iran
National Math Olympiad (3rd Round)
2006

Linear Algebra

1 Suppose that A Mn (R) with Rank(A) = k. Prove that A is sum of k matrices X1 , . . . , Xk


with Rank(Xi ) = 1.
2 f : Rn Rm is a non-zero linear map. Prove that there is a base {v1 , . . . , vn m} for Rn that
the set {f (v1 ), . . . , f (vn )} is linearly independent, after ommitting Repetitive elements.
3 Suppose (u, v) is an inner product on Rn and f : Rn Rn is an isometry, that f (0) = 0. 1)
Prove that for each u, v we have (u, v) = (f (u), f (v) 2) Prove that f is linear.
4 f : Rn Rn is a bijective map, that Image of every n 1-dimensional affine space is a
n 1-dimensional affine space. 1) Prove that Image of every line is a line. 2) Prove that f is
an affine map. (i.e. f = goh that g is a translation and h is a linear map.)

This file was downloaded from the AoPS Math Olympiad Resources Page
http://www.artofproblemsolving.com/

Page 6

iran
National Math Olympiad (3rd Round)
2006

Number Theory

1 n is a natural number. d is the least natural number that for each a that gcd(a, n) = 1 we
know ad 1 (mod n). Prove that there exist a natural number that ordn b = d
n

+1
2 n is a natural number that xx+1
is irreducible over Z2 [x]. Consider a vector in Zn2 that it has
odd number of 1s (as entries) and at least one of its entries are 0. Prove that these vector
and its translations are a basis for Zn2

3 L is a fullrank lattice in R2 and K is a sub-lattice of L, that A(K)


A(L) = m. If m is the least
number that for each x L, mx is in K. Prove that there exists a basis {x1 , x2 } for L that
{x1 , mx2 } is a basis for K.
4 a, b, c, t are antural numbers and k = ct and n = ak bk . a) Prove that if k has at least q
different prime divisors, then n has at least qt different prime divisors. b)Prove that (n) id
t
divisible by 2 2
5 For each n, suppsoe L(n) is the number of natural numbers 1 a n that n|an 1. If
p1 , p2 , . . . , pk are prime divisors of n, define T (n) as (p1 1)(p2 1) . . . (pk 1). a) Prove that
for each n N
n|L(n)T (n)
b)Prove that if gcd(n, T (n)) = 1 then (n) = L(n)T (n)
6 a) P (x), Q(x) are polynomials with rational coefficients and P (x) is not the zero polynomial.
Prove that there exist a non-zero polynomial Q(x) Q[x] that
P (x)|Q(R(x))
b) P, Q are polynomial with integer coefficients and P is monic. Prove that there exist a
monic polynomial Q(x) Z[x] that
P (x)|Q(R(x))

This file was downloaded from the AoPS Math Olympiad Resources Page
http://www.artofproblemsolving.com/

Page 7

iran
National Math Olympiad (3rd Round)
2007

AlgebraAnalysis

1 Let a, b be two complex numbers. Prove that roots of z 4 + az 2 + b form a rhombus with origin
2
as center, if and only if ab is a non-positive real number.
2 a, b, c are three different positive real numbers. Prove that:


a + b b + c c + a


a b + b c + c a > 1
3 Find the largest real T such that for each non-negative real numbers a, b, c, d, e such that
a + b = c + d + e:
p

a2 + b2 + c2 + d2 + e2 T ( a + b + c + d + e)2
4 a) Let n1 , n2 , . . . be a sequence of natural number such that ni 2 and 1 , 2 , . . . be a
sequence such that i {1, 2}. Prove that the sequence:
r
q

n1
1 + n2 2 + + nk k
p

is convergent and its limit is in (1, 2]. Define n1 1 + n2 2 + . . . to be this limit. b) Prove
that for each x (1, 2] there exist sequences
n1 , n2 , N and ni 2 and 1 , 2 , . . . , such
p

that ni 2 and i {1, 2}, and x = n1 1 + n2 2 + . . .


5 Prove that for two non-zero polynomials f (x, y), g(x, y) with real coefficients the system:

f (x, y) = 0
g(x, y) = 0
has finitely many solutions in C2 if and only if f (x, y) and g(x, y) are coprime.

This file was downloaded from the AoPS Math Olympiad Resources Page
http://www.artofproblemsolving.com/

Page 1

iran
National Math Olympiad (3rd Round)
2007

Final Exam

1 Consider two polygons P and Q. We want to cut P into some smaller polygons and put
them together in such a way to obtain Q. We can translate the pieces but we can not rotate
them or reflect them. We call P, Q equivalent if and only if we can obtain Q from P (which is
obviously an equivalence relation). [img]http://i3.tinypic.com/4lrb43k.png[/img] a) Let P, Q
be two rectangles with the same area(their sides are not necessarily parallel). Prove that
P and Q are equivalent. b) Prove that if two triangles are not translation of each other,
they are not equivalent. c) Find a necessary and sufficient condition for polygons P, Q to be
equivalent.
2 We call the mapping : Z\{0} N, a degree mapping if and only if for each a, b Z such
that b 6= 0 and b 6 |a there exist integers r, s such that a = br + s, and (s) < (b). a) Prove
that the following mapping is a degree mapping:
(n) = Number of digits in the binary representation of n
b) Prove that there exist a degree mapping 0 such that for each degree mapping and for
each n 6= 0, 0 (n) (n). c) Prove that = 0 [img]http://i16.tinypic.com/4qntmd0.png[/img]
3 We call a set A a good set if it has the following properties: 1. A consists circles in plane. 2.
No two element of A intersect. Let A, B be two good sets. We say A, B are equivalent if we can
reach from A to B by moving circles in A, making them bigger or smaller in such a way that
during these operations each circle does not intersect with other circles. Let an be the number
of inequivalent good subsets with n elements. For example a1 = 1, a2 = 2, a3 = 4, a4 = 9.
[img]http://i5.tinypic.com/4r0x81v.png[/img] If there exist a, b such that Aan an Bbn ,
we say growth ratio of an is larger than a and is smaller than b. a) Prove that growth ratio of
an is larger than 2 and is smaller than 4. b) Find better bounds for upper and lower growth
ratio of an .
4 In the following triangular lattice distance of two vertices is length of the shortest path between
them. Let A1 , A2 , . . . , An be constant vertices of the lattice. We want to find a vertex in the
lattice whose sum of distances from vertices is minimum. We start from an arbitrary vertex.
At each step we check all six neighbors and if sum of distances from vertices of one of the
neighbors is less than sum of distances from vertices at the moment we go to that neighbor.
If we have more than one choice we choose arbitrarily, as seen in the attached picture.
Obviusly the algorithm finishes a) Prove that when we can not make any move we have
reached to the problems answer. b) Does this algorithm reach to answer for each connected
graph?

This file was downloaded from the AoPS Math Olympiad Resources Page
http://www.artofproblemsolving.com/

Page 2

iran
National Math Olympiad (3rd Round)
2007

5 Look at these fractions. At firs step we have


a
c
b and d , and we do this forever
0
1
0
1
0
1
0
1

1
3

1
2
1
2

2
3

0
1

and 10 , and at each step we write

1
1
1
1
1
1

3
2

2
1
2
1

3
1

a+b
c+d

between

1
0
1
0
1
0
1
0

...

a) Prove that each of these fractions is irreducible. b) In the plane we have put infinitely
many circles of diameter 1, over each integer on the real line, one circle. The inductively
we put circles that each circle is tangent to two adjacent circles and real line, and we
do this forever. Prove that points of tangency of these circles are exactly all the numbers in part a(except 01 ). [img]http://i2.tinypic.com/4m8tmbq.png[/img] c) Prove that in
these two parts all of positive rational numbers appear. If you dont understand the numbers, look at [url=http://upload.wikimedia.org/wikipedia/commons/2/21/Arabicn umerals
en.svg]here[/url].Scientisthavesucceededtof indnewnumbersbetweenrealnumberswithstrongmicroscopes.N
R), andalso4operationsaddition, multiplication, ...andtheseoperationhaveallpropertiesthesameas
R.[img]http : //i14.tinypic.com/4tk6mnr.png[/img]a)P rovethatinthislargersystemthereisanumberwhichis
R[x].
67 A ring is the area between two circles with the same center, and width of a ring is the difference
between the radii of two circles. [img]http://i18.tinypic.com/6cdmvi8.png[/img] a) Can we put
uncountable disjoint rings of width 1(not necessarily same) in the space such that each two of them
can not be separated. [img]http://i19.tinypic.com/4qgx30j.png[/img] b) Whats the answer if 1 is
replaced with 0?
8 In this question you must make all numbers of a clock, each with using 2, exactly 3 times and
Mathematical symbols. You are not allowed to use English alphabets and words like sin or lim or
a, b and no other digits. [img]http://i2.tinypic.com/5x73dza.png[/img]

This file was downloaded from the AoPS Math Olympiad Resources Page
http://www.artofproblemsolving.com/

Page 3

iran
National Math Olympiad (3rd Round)
2007

Geometry

1 Let ABC, l and P be arbitrary triangle, line and point. A0 , B 0 , C 0 are reflections of A, B, C
in point P . A00 is a point on B 0 C 0 such that AA00 k l. B 00 , C 00 are defined similarly. Prove
that A00 , B 00 , C 00 are collinear.
2 a) Let ABC be a triangle, and O be its circumcenter. BO and CO intersect with AC, AB at
B 0 , C 0 . B 0 C 0 intersects the circumcircle at two points P, Q. Prove that AP = AQ if and only
if ABC is isosceles. b) Prove the same statement if O is replaced by I, the incenter.
3 Let I be incenter of triangel ABC, M be midpoint of side BC, and T be the intersection point
of IM with incircle, in such a way that I is between M and T . Prove that BIM CIM =
3
2 (B C), if and only if AT BC.
4 Let ABC be a triangle, and D be a point where incircle touches side BC. M is midpoint
of BC, and K is a point on BC such that AK BC. Let D0 be a point on BC such that
D0 M
DM
0
D0 K = DK . Define a to be circle with diameter DD . We define B ,C similarly. Prove that
every two of these circles are tangent.
5 Let ABC be a triangle. Squares ABc Ba C, CAb Ac B and BCa Cb A are outside the triangle.
Square Bc Bc0 Ba0 Ba with center P is outside square ABc Ba C. Prove that BP, Ca Ba and Ac Bc
are concurrent.

This file was downloaded from the AoPS Math Olympiad Resources Page
http://www.artofproblemsolving.com/

Page 4

iran
National Math Olympiad (3rd Round)
2007

Number Theory

1 Let n be a natural number, such that (n, 2(21386 1)) = 1. Let {a1 , a2 , . . . , a(n) } be a reduced
residue system for n. Prove that:
n|a1386
+ a1386
+ + a1386
1
2
(n)
2 Let m, n be two integers such that (m) = (n) = c. Prove that there exist natural numbers
b1 , b2 , . . . , bc such that {b1 , b2 , . . . , bc } is a reduced residue system with both m and n.
3 Let n be a natural number, and n = 22007 k + 1, such that k is an odd number. Prove that
n 6 |2n1 + 1
4 Find all integer solutions of
x4 + y 2 = z 4
5 A hyper-primitive root is a k-tuple (a1 , a2 , . . . , ak ) and (m1 , m2 , . . . , mk ) with the following
property: For each a N, that (a, m) = 1, has a unique representation in the following form:
a a1 1 a2 2 . . . ak k

(mod m)

1 i mi

Prove that for each m we have a hyper-primitive root.

6 Something related to this [url=http://www.mathlinks.ro/Forum/viewtopic.php?p=845756845756]problem[/u


Pn
i
Prove that for a set S N, there exists a sequence {ai }
i=0 in S such that for each n,
i=0 ai x
is irreducible in Z[x] if and only if |S| 2.
By Omid Hatami

This file was downloaded from the AoPS Math Olympiad Resources Page
http://www.artofproblemsolving.com/

Page 5

iran
National Math Olympiad (3rd Round)
2008

Algebra

1 Suppose that f (x) Z[x] be an irreducible polynomial. It is known that f has a root of norm
larger than 32 . Prove that if is a root of f then f (3 + 1) 6= 0.
2 Find the smallest real K such that for each x, y, z R+ :
p

x y + y z + z x K (x + y)(y + z)(z + x)
3 Let (b0 , b1 , b2 , b3 ) be a permutation of the set {54, 72, 36, 108}. Prove that x5 + b3 x3 + b2 x2 +
b1 x + b0 is irreducible in Z[x].
4 Let x, y, z R+ and x + y + z = 3. Prove that:
x3
2
y3
z3
1
+
+
+ (xy + xz + yz)
3
3
3
y +8 z +8 x +8
9 27
5 Prove that the following polynomial is irreducible in Z[x, y]:
x200 y 5 + x51 y 100 + x106 4x100 y 5 + x100 2y 100 2x6 + 4y 5 2

This file was downloaded from the AoPS Math Olympiad Resources Page
http://www.artofproblemsolving.com/

Page 1

iran
National Math Olympiad (3rd Round)
2008

Combinatorics

1 Prove that the number of pairs (, S) of a permutation of {1, 2, . . . , n} and a subset S of


{1, 2, . . . , n} such that
x S : (x) 6 S
is equal to n!Fn+1 in which Fn is the Fibonacci sequence such that F1 = F2 = 1
2 Prove that the number permutations of {1, 2, . . . , n} s.t. there does not exist i < j < n s.t.
(i) < (j + 1) < (j) is equal to the number of partitions of that set.
3 Prove that for each n:

n 
X
n+k1
k=1

2k 1

= F2n

4 Let S be a sequence that:

S0 = 0
S1 = 1

Sn = Sn1 + Sn2 + Fn (n > 1)


such that Fn is Fibonacci sequence such that F1 = F2 = 1. Find Sn in terms of Fibonacci
numbers.
5 n people decide to play a game. There are n 1 ropes and each of its two ends are in hand of
one of the players, in such a way that ropes and players form a tree. (Each person can hold
more than rope end.)
At each step a player gives one of the rope ends he is holding to another player. The goal is
to make a path of length n 1 at the end.
But the game regulations change before game starts. Everybody has to give one of his rope
ends only two one of his neighbors. Let a and b be minimum steps for reaching to goal in
these two games. Prove that a = b if and only if by removing all players with one rope end
(leaves of the tree) the remaining people are on a path. (the remaining graph is a path.)
[img]http://i37.tinypic.com/2l9h1tv.png[/img]

This file was downloaded from the AoPS Math Olympiad Resources Page
http://www.artofproblemsolving.com/

Page 2

iran
National Math Olympiad (3rd Round)
2008

Complex numbers

has a root on
1 Prove that for n > 0 and a 6= 0 the polynomial p(z) = az 2n+1 + bz 2n + bz + a
unit circle
2 Let g, f : C C be two continuous functions such that for each z 6= 0, g(z) = f ( z1 ). Prove
that there is a z C such that f ( z1 ) = f (
z)
3 For each c C, let fc (z, 0) = z, and fc (z, n) = fc (z, n 1)2 + c for n 1. a) Prove that
if |c| 14 then there is a neighborhood U of origin such that for each z U the sequence
fc (z, n), n N is bounded. b) Prove that if c > 14 is a real number there is a neighborhood U
of origin such that for each z U the sequence fc (z, n), n N is unbounded.

This file was downloaded from the AoPS Math Olympiad Resources Page
http://www.artofproblemsolving.com/

Page 3

iran
National Math Olympiad (3rd Round)
2008

Final Exam

1 Police want to arrest on the famous criminals of the country whose name is Kaiser. Kaiser
is in one of the streets of a square shaped city with n vertical streets and n horizontal streets. In the following cases how many police officers are needed to arrest Kaiser?
[img]http://i38.tinypic.com/2i1icect h.png[/img][img]http : //i34.tinypic.com/28rk4s3t h.png[/img]a)Eachpo
//i38.tinypic.com/35n615y.png[/img]
23 a) Prove that there are two polynomials in Z[x] with at least one coefficient larger than 1387 such
that coefficients of their product is in the set {1, 0, 1}. b) Does there exist a multiple of x2 3x+1
such that all of its coefficient are in the set {1, 0, 1}
4 =A subset S of R2 is called an algebraic set if and only if there is a polynomial p(x, y) R[x, y]
such that
S = {(x, y) R2 |p(x, y) = 0}
Are the following subsets of plane an algebraic sets? 1. A square [img]http://i36.tinypic.com/28uiaep.png[/img]
2. A closed half-circle [img]http://i37.tinypic.com/155m155.png[/img]
5 a) Suppose that RBR0 B 0 is a convex quadrilateral such that vertices R and R0 have red color
and vertices B and B 0 have blue color. We put k arbitrary points of colors blue and red in the
quadrilateral such that no four of these k + 4 point (except probably RBR0 B 0 ) lie one a circle.
Prove that exactly one of the following cases occur? 1. There is a path from R to R0 such that
distance of every point on this path from one of red points is less than its distance from all blue
points. 2. There is a path from B to B 0 such that distance of every point on this path from one of
blue points is less than its distance from all red points. We call these two paths the blue path and
the red path respectively.
Let n be a natural number. Two people play the following game. At each step one player puts a
point in quadrilateral satisfying the above conditions. First player only puts red point and second
player only puts blue points. Game finishes when every player has put n points on the plane. First
players goal is to make a red path from R to R0 and the second players goal is to make a blue
path from B to B 0 . b) Prove that if RBR0 B 0 is rectangle then for each n the second player wins.
c) Try to specify the winner for other quadrilaterals.
6 There are five research labs on Mars. Is it always possible to divide Mars to five connected congruent
regions such that each region contains exactly on research lab. [img]http://i37.tinypic.com/f2iq8g.png[/img]
7 A graph is called a self-interesting graph if and only if it is isomorphic to a graph whose every edge is a segment and every two edges intersect. Notice that no edge contains a vertex
except its two endings. a) Find all ns for which the cycle of length n is self-intersecting. b)

This file was downloaded from the AoPS Math Olympiad Resources Page
http://www.artofproblemsolving.com/

Page 4

iran
National Math Olympiad (3rd Round)
2008

Prove that in a self-intersecting graph |E(G)| |V (G)|. c) Find all self-intersecting graphs.
[img]http://i35.tinypic.com/x43s5u.png[/img]
8 In an old script found in ruins of Perspolis is written: [code] This script has been finished in a year
whose 13th power is 258145266804692077858261512663 You should know that if you are skilled in
Arithmetics you will know the year this script is finished easily.[/code] Find the year the script is
finished. Give a reason for your answer.

This file was downloaded from the AoPS Math Olympiad Resources Page
http://www.artofproblemsolving.com/

Page 5

iran
National Math Olympiad (3rd Round)
2008

Geometry

1 Let ABC be a triangle with BC > AC > AB. Let A0 , B 0 , C 0 be feet of perpendiculars from
A, B, C to BC, AC, AB, such that AA0 = BB 0 = CC 0 = x. Prove that: a) If ABC A0 B 0 C 0
then x = 2r b) Prove that if A0 , B 0 and C 0 are collinear, then x = R + d or x = R d.
(In this problem R is the radius of circumcircle, r is radius of incircle and d = OI)
2 Let la , lb , lc be three parallel lines passing through A, B, C respectively. Let la0 be reflection
of la into BC. lb0 and lc0 are defined similarly. Prove that la0 , lb0 , lc0 are concurrent if and only if
la is parallel to Euler line of triangle ABC.
3 Let ABCD be a quadrilateral, and E be intersection points of AB, CD and AD, BC respectively. External bisectors of DAB and DCB intersect at P , external bisectors of ABC and
ADC intersect at Q and external bisectors of AED and AF B intersect at R. Prove that
P, Q, R are collinear.
4 Let ABC be an isosceles triangle with AB = AC, and D be midpoint of BC, and E be
foot of altitude from C. Let H be orthocenter of ABC and N be midpoint of CE. AN
intersects with circumcircle of triangle ABC at K. The tangent from C to circumcircle of
ABC intersects with AD at F . Suppose that radical axis of circumcircles of CHA and CKF
is BC. Find BAC.
5 Let D, E, F be tangency point of incircle of triangle ABC with sides BC, AC, AB. DE and
DF intersect the line from A parallel to BC at K and L. Prove that the Euler line of triangle
DKL passes through Feuerbach point of triangle ABC.

This file was downloaded from the AoPS Math Olympiad Resources Page
http://www.artofproblemsolving.com/

Page 6

iran
National Math Olympiad (3rd Round)
2008

Number Theory

1 Let k > 1 be an integer. Prove that there exists infinitely many natural numbers such as n
such that:
n|1n + 2n + + k n
2 Prove that there exists infinitely many primes p such that:
13|p3 + 1
3 Let P be a regular polygon. A regular sub-polygon of P is a subset of vertices of P with at
least two vertices such that divides the circumcircle to equal arcs. Prove that there is a subset
of vertices of P such that its intersection with each regular sub-polygon has even number of
vertices.
4 Let u be an odd number. Prove that

33u 1
3u 1

can be written as sum of two squares.

5 Find all polynomials f Z[x] such that for each a, b, x N


a + b + c|f (a) + f (b) + f (c)

This file was downloaded from the AoPS Math Olympiad Resources Page
http://www.artofproblemsolving.com/

Page 7

iran
National Math Olympiad (3rd Round)
2010

Day 1
1 suppose that polynomial p(x) = x2010 x2009 ... x 1 does not have a real root. what is
the maximum number of coefficients to be 1?(14 points)
2 a, b, c are positive real numbers. prove the following inequality:
1
a2

1
b2

1
c2

1
(a+b+c)2

7 1
25 ( a

1
b

1
c

1
2
a+b+c )

(20 points)
3 prove that for each natural number n there exist a polynomial with degree 2n + 1 with
coefficients in Q[x] such that it has exactly 2 complex zeros and its irreducible in Q[x].(20
points)
4 For each polynomial p(x) = an xn + an1 xn1 + ... + a1 x + a0 we define its derivative as this
and we show it by p0 (x):
p0 (x) = nan xn1 + (n 1)an1 xn2 + ... + 2a2 x + a1
a) For each two polynomials p(x) and q(x) prove that:(3 points)
(p(x)q(x))0 = p0 (x)q(x) + p(x)q 0 (x)
b) Suppose that p(x) is a polynomial with degree n and x1 , x2 , ..., xn are its zeros. prove
that:(3 points)
n
p0 (x) X 1
=
p(x)
x xi
i=1

c) p(x) is a monic polynomial with degree n and z1 , z2 , ..., zn are its zeros such that:
|z1 | = 1,

i {2, .., n} : |zi | 1

Prove that p0 (x) has at least one zero in the disc with length one with the center z1 in complex
plane. (disc with length one with the center z1 in complex plane: D = {z C : |z z1 |
1})(20 points)

2
2
2
5 x, y, z are positive real numbers such that xy +yz +zx = 1. prove that: 3 3+ xy + yz + zx
(x + y + z)2 (20 points)
the exam time was 6 hours.

This file was downloaded from the AoPS Math Olympiad Resources Page
http://www.artofproblemsolving.com/

Page 1

iran
National Math Olympiad (3rd Round)
2010

Day 2
1 suppose that a = 3100 and b = 5454. how many zs in [1, 399 ) exist such that for every c that
gcd(c, 3) = 1, two equations xz c and xb c (mod a) have the same number of answers?( 100
6
points)
2 R is a ring such that xy = yx for every x, y R and if ab = 0 then a = 0 or b = 0. if for every
Ideal I R there exist x1 , x2 , .., xn in R (n is not constant) such that I = (x1 , x2 , ..., xn ),
prove that every element in R that is not 0 and its not a unit, is the product of finite
irreducible elements.( 100
6 points)
3 If p is a prime number, what is the product of elements like g such that 1 g p2 and g is
a primitive root modulo p but its not a primitive root modulo p2 , modulo p2 ?( 100
6 points)
P
4 sppose that k : N R is a function such that k (n) = d|n dk . k : N R is a function
such that k k = . find a formula for k .( 100
6 points)
5 prove that if p is a prime number such that p = 12k + {2, 3, 5, 7, 8, 11}(k N {0}), there
exist a field with p2 elements.( 100
6 points)
6 g and n are natural numbers such that gcd(g 2 g, n) = 1 and A = {g i |i N} and B = {x
(n)|x A}(by x (n) we mean a number from the set {0, 1, ..., n 1} which is congruent
P
n(i+1)
) B| prove that g 1| g1
with x modulo n). if for 0 i g 1 ai = |[ ni
i=0 iai .( the
g ,
g
100
symbol | | means the number of elements of the set)( 6 points)
the exam time was 4 hours

This file was downloaded from the AoPS Math Olympiad Resources Page
http://www.artofproblemsolving.com/

Page 2

iran
National Math Olympiad (3rd Round)
2010

Day 3
1 1. In a triangle ABC, O is the circumcenter and I is the incenter. X is the reflection of I
to O. A1 is foot of the perpendicular from X to BC. B1 and C1 are defined similarly. prove
that AA1 ,BB1 and CC1 are concurrent.(12 points)
2 in a quadrilateral ABCD, E and F are on BC and AD respectively such that the area of
triangles AED and BCF is 47 of the area of ABCD. R is the intersection point of digonals
AR
BR
of ABCD. RC
= 35 and RD
= 56 . a) in what ratio does EF cut the digonals?(13 points) b)
find FAF
D .(5 points)
3 in a quadrilateral ABCD digonals are perpendicular to each other. let S be the intersection
of digonals. K,L,M and N are reflections of S to AB,BC,CD and DA. BN cuts the
circumcircle of SKN in E and BM cuts the circumcircle of SLM in F . prove that EF LK
is concyclic.(20 points)
4 in a triangle ABC, I is the incenter. BI and CI cut the circumcircle of ABC at E and F
respectively. M is the midpoint of EF . C is a circle with diameter EF . IM cuts C at two
points L and K and the arc BC of circumcircle of ABC (not containing A) at D. prove that
DL
DK
IL = IK .(25 points)
5 In a triangle ABC, I is the incenter. D is the reflection of A to I. the incircle is tangent to
BC at point E. DE cuts IG at P (G is centroid). M is the midpoint of BC. prove that a)
AP ||DM .(15 points) b) AP = 2DM . (10 points)
6 In a triangle ABC, C = 45. AD is the altitude of the triangle. X is on AD such that
XBC = 90 B (X is in the triangle). AD and CX cut the circumcircle of ABC in M
and N respectively. if tangent to circumcircle of ABC at M cuts AN at P , prove that P ,B
and O are collinear.(25 points)
the exam time was 4 hours and 30 minutes.

This file was downloaded from the AoPS Math Olympiad Resources Page
http://www.artofproblemsolving.com/

Page 3

iran
National Math Olympiad (3rd Round)
2010

Day 4
1 suppose that F X (k) and |X| = n. we know that for every three distinct elements of
F like A, B, C, at mostone of A B,B C and C A is . for k n2 prove that: a)
n1
|F| max(1, 4 nk )
.(15 points) b) find all cases of equality in a) for k n3 .(5
k1
points)
S
2 suppose that F nj=k+1 X (j) and |X| = n. we know that F is a sperner family and its also
P
Hk . prove that: BF  1  1 (15 points)
n1
|B| 1
3 suppose that F p(X) and |X| = n. we knowthatfor every Ai , Aj F that Ai Aj we
n
n
have 3 |Ai | |Aj |. prove that: |F| b 23 + 21 n c (20 points)
b2c
4 suppose that F X (K) and |X| = n. we know that for every three distinct elements of F
like A, B and C we have A B 6 C.
a)(10 points) Prove that :

|F|

k
b k2 c


+1

b)(15 points) if elements of F do not necessarily have k elements, with the above conditions
show that:


n
|F|
+2
d n2
3 e
 
Pk1 n
5 suppose that F p(X) and |X| = n. prove that if |F| > i=0
then there exist Y X
i
with |Y | = k such that p(Y ) = F Y that F Y = {F Y : F F}(20 points) you can
see this problem also here: COMBINATORIAL PROBLEMS AND EXERCISES-SECOND
EDITION-by LASZLO LOVASZ-AMS CHELSEA PUBLISHING- chapter 13- problem 10(c)!!!
6 Suppose that X is a set with n elements and F X (k) and X1 , X2 , ..., Xs is a partition of X.
S
S
we know that for every A, B F and every 1 j s, E = B( ji=1 Xi ) 6= A( ji=1 Xi ) = F
shows that non of E, F have the other one. prove that:

s 
Y
|X
|
i
|F| maxPS wi =k
i=1
wi
i=1

(15 points)
the exam time was 5 hours and 20 minutes.

This file was downloaded from the AoPS Math Olympiad Resources Page
http://www.artofproblemsolving.com/

Page 4

iran
National Math Olympiad (3rd Round)
2010

Day 5
1 two variable ploynomial
P (x, y) is a two variable polynomial with real coefficients. degree of a monomial means sum
of the powers of x and y in it. we denote by Q(x, y) sum of monomials with the most degree
in P (x, y). (for example if P (x, y) = 3x4 y 2x2 y 3 +5xy 2 +x5 then Q(x, y) = 3x4 y 2x2 y 3 .)
suppose that there are real numbers x1 ,y1 ,x2 and y2 such that Q(x1 , y1 ) > 0 , Q(x2 , y2 ) < 0
prove that the set {(x, y)|P (x, y) = 0} is not bounded. (we call a set S of plane bounded if
there exist positive number M such that the distance of elements of S from the origin is less
than M .)
time allowed for this question was 1 hour.
2 rolling cube
a,b and c are natural numbers. we have a (2a + 1) (2b + 1) (2c + 1) cube. this cube is
on an infinite plane with unit squares. you call roll the cube to every side you want. faces of
the cube are divided to unit squares and the square in the middle of each face is coloured (it
means that if this square goes on a square of the plane, then that square will be coloured.)
prove that if any two of lengths of sides of the cube are relatively prime, then we can colour
every square in plane.
time allowed for this question was 1 hour.
3 points in plane
set A containing n points in plane is given. a copy of A is a set of points that is made by
using transformation, rotation, homogeneity or their combination on elements of A. we want
to put n copies of A in plane, such that every two copies have exactly one point in common
and every three of them have no common elements. a) prove that if no 4 points of A make
a parallelogram, you can do this only using transformation. (A doesnt have a parallelogram
with angle 0 and a parallelogram that its two non-adjacent vertices are one!) b) prove that
you can always do this by using a combination of all these things.
time allowed for this question was 1 hour and 30 minutes
4 carpeting
suppose that S is a figure in the plane such that its border doesnt contain any lattice points.
suppose that x, y are two lattice points with the distance 1 (we call a point lattice point if
its coordinates are integers). suppose that we can cover the plane with copies of S such that
x, y always go on lattice points ( you can rotate or reverse copies of S). prove that the area
of S is equal to lattice points inside it.
time allowed for this question was 1 hour.

This file was downloaded from the AoPS Math Olympiad Resources Page
http://www.artofproblemsolving.com/

Page 5

iran
National Math Olympiad (3rd Round)
2010

5 interesting sequence
n is a natural number and x1 , x2 , ... is a sequence of numbers 1 and 1 with these properties:
it is periodic and its least period number is 2n 1. (it means that for every natural number
j we have xj+2n 1 = xj and 2n 1 is the least number with this property.)
There exist distinct integers 0 t1 < t2 < ... < tk < n such that for every natural number j
we have
xj+n = xj+t1 xj+t2 ... xj+tk
Prove that for every natural number s that s < 2n 1 we have
n 1
2X

xi xi+s = 1

i=1

Time allowed for this question was 1 hours and 15 minutes.


6 polyhedral
we call a 12-gon in plane good whenever: first, it should be regular, second, its inner plane
must be filled!!, third, its center must be the origin of the coordinates, forth, its vertices
must have points (0, 1),(1, 0),(1, 0) and (0, 1). find the faces of the massivest polyhedral
that its image on every three plane xy,yz and zx is a good 12-gon. (its obvios that centers
of these three 12-gons are the origin of coordinates for three dimensions.)
time allowed for this question is 1 hour.
7 interesting function
S is a set with n elements and P (S) is the set of all subsets of S and f : P (S) N is a
function with these properties: for every subset A of S we have f (A) = f (S A). for every
two subsets of S like A and B we have max(f (A), f (B)) f (A B) prove that number of
natural numbers like x such that there exists A S and f (A) = x is less than n.
time allowed for this question was 1 hours and 30 minutes.
8 numbers n2 + 1
prove that there infinitly many natural numbers in the form n2 + 1 such that they dont have
any divider in the form of k 2 + 1 except 1 and itself.
time allowed for this question was 45 minutes.

This file was downloaded from the AoPS Math Olympiad Resources Page
http://www.artofproblemsolving.com/

Page 6

iran
National Math Olympiad (3rd Round)
2010

Day 6
1 prove that the group of oriention-preserving symmetries of a cube is isomorph to S4 (group of
permutations of {1, 2, 3, 4}).(20 points)
2 prove the third sylow theorem: suppose that G is a group and |G| = pe m which p is a prime
number and (p, m) = 1. suppose that a is the number of p-sylow subgroups of G (H < G that
|H| = pe ). prove that a|m and p|a 1.(Hint: you can use this: every two p-sylow subgroups
are conjugate.)(20 points)
3 suppose that G < Sn is a subgroup of permutations of {1, ..., n} with this property that for
every e 6= g G there exist exactly one k {1, ..., n} such that g.k = k. prove that there
exist one k {1, ..., n} such that for every g G we have g.k = k.(20 points)
4 a) prove that every discrete subgroup of (R2 , +) is in one of these forms: i-{0}. ii-{mv|m Z}
for a vector v in R2 . iii-{mv + nw|m, n Z} for tho linearly independent vectors v and w
in R2 .(lattice L) b) prove that every finite group of symmetries that fixes the origin and the
lattice L is in one of these forms: Ci or Di that i = 1, 2, 3, 4, 6 (Ci is the cyclic group of order
i and Di is the dyhedral group of order i).(20 points)
5 suppose that p is a prime number. find that smallest n such that there exists a non-abelian
group G with |G| = pn .
SL is an acronym for Special Lesson. this year our special lesson was Groups and Symmetries.
the exam time was 5 hours.

This file was downloaded from the AoPS Math Olympiad Resources Page
http://www.artofproblemsolving.com/

Page 7

iran
Team Selection Test
2002

1 ABCD is a convex quadrilateral. We draw its diagnals to divide the quadrilateral to four triabgles. P is the intersection of diagnals. I1 , I2 , I3 , I4 are excenters of P AD, P AB, P BC, P CD(excenters
corresponding vertex P ). Prove that I1 , I2 , I3 , I4 lie on a circle iff ABCD is a tangential
quadrilateral.
2 n people (with names 1, 2, . . . , n) are around a table. Some of them are friends. At each step 2
friend can change their place. Find a necessary and sufficient condition for friendship relation
between them that with these steps we can always reach to all of posiible permutations.
3 A 2-line is the area between two parallel lines. Length of 2-line is distance of two parallel
lines. We have covered unit circle with some 2-lines. Prove sum of lengths of 2-lines is
at least 2.
4 O is a point in triangle ABC. We draw perpendicular from O to BC, AC, AB which intersect
BC, AC, AB at A1 , B1 , C1 . Prove that O is circumcenter of triangle ABC iff perimeter of
ABC is not less than perimeter of triangles AB1 C1 , BC1 A1 , CB1 A1 .
5 A school has n students and k classes. Every two students in the same class are friends. For
each two different classes, there are two people from these classes that are not friends. Prove
that we can divide students into n k + 1 parts taht students in each part are not friends.
Pn
Pn
+
2
6 Assume
l 2 x1 , x22 ,m. . . , xn R , i=1 xi = n, i=1 xi s > 0 and 0 1. Prove that at
of these numbers are larger than s
least s (1)
n
n.
7 S1 , S2 , S3 are three spheres in R3 that their centers are not collinear. k 8 is the number
of planes that touch three spheres. Ai , Bi , Ci is the point that i-th plane touch the spheres
S1 , S2 , S3 . Let Oi be circumcenter of Ai Bi Ci . Prove that Oi are collinear.
8 We call A1 , A2 , A3 mangool iff there is a permutation that A(2) 6 A(1) , A(3) 6 A(1)
A(2) . A good family is a family of finite subsets of N like X, A1 , A2 , . . . , An . To each goo
family we correspond a graph with vertices {A1 , A2 , . . . , An }. Connect Ai , Aj iff X, Ai , Aj are
mangool sets. Find all graphs that we can find a good family corresponding to it.
9 (n) is the number of primes that are not bigger than n. For n = 2, 3, 4, 6, 8, 33, . . . we have
(n)|n. Does exist infinitely many integers n that (n)|n?
10 Suppose from (m + 2) (n + 2) rectangle we cut 4, 1 1 corners. Now on first and last row
first and last columns we write 2(m + n) real numbers. Prove we can fill the interior m n
rectangle with real numbers that every number is average of its 4 neighbors.
11 A 10 10 10 cube has 1000 unit cubes. 500 of them are coloured black and 500 of them
are coloured white. Show that there are at least 100 unit squares, being the common face of
a white and a black unit cube.

This file was downloaded from the AoPS Math Olympiad Resources Page
http://www.artofproblemsolving.com/

Page 1

iran
Team Selection Test
2002

12 We call a permutation (a1 , a2 , ..., an ) of (1, 2, ..., n) quadratic if there exists at least a perfect
square among the numbers a1 , a1 + a2 , ..., a1 + a2 + ... + an . Find all natural numbers n such
that all permutations in Sn are quadratic.
Remark. Sn denotes the n-th symmetric group, the group of permutations on n elements.
13 Let ABC be a triangle. The incircle of triangle ABC touches the side BC at A0 , and the
line AA0 meets the incircle again at a point P . Let the lines CP and BP meet the incircle of
triangle ABC again at N and M , respectively. Prove that the lines AA0 , BN and CM are
concurrent.

This file was downloaded from the AoPS Math Olympiad Resources Page
http://www.artofproblemsolving.com/

Page 2

iran
Team Selection Test
2004

1 Suppose that p is a prime number. Prove that for each k, there exists an n such that:


n
p


=

n+k
p

2 Suppose that p is a prime number. Prove that the equation x2 py 2 = 1 has a solution if
and only if p 1 (mod 4).
3 Suppose that ABCD is a convex quadrilateral. Let F = AB CD, E = AD BC and
T = AC BD. Suppose that A, B, T, E lie on a circle which intersects with EF at P . Prove
that if M is midpoint of AB, then AP M = BP T .
4 Let M, M 0 be two conjugates point in triangle ABC (in the sense that M AB = M 0 AC, . . . ).
Let P, Q, R, P 0 , Q0 , R0 be foots of perpendiculars from M and M 0 to BC, CA, AB. Let
E = QR Q0 R0 , F = RP R0 P 0 and G = P Q P 0 Q0 . Prove that the lines AG, BF, CE are
parallel.
5 This problem is generalization of [url=http://www.mathlinks.ro/Forum/viewtopic.php?t=5918]this
one[/url]. Suppose G is a graph and S V (G). Suppose we have arbitrarily assign real numbers to each element of S. Prove that we can assign numbers to each vertex in G\S that for
each v G\S number assigned to v is average of its neighbors.
6 p is a polynomial with integer coefficients and for every natural n we have p(n) > n xk is a sequence
that: x1 = 1, xi+1 = p(xi )for every N one of xi is divisible by N Prove p(x) = x + 1

N opain,nogain

This file was downloaded from the AoPS Math Olympiad Resources Page
http://www.artofproblemsolving.com/

Page 1

iran
Team Selection Test
2005

Day 1
1 Suppose that a1 , a2 , ..., an are positive real numbers such that a1 a2 an . Let
a1 + a2 + + an
= m;
n

a21 + a22 + + a2n


= 1.
n

Suppose that, for some i, we know ai m. Prove that:


n i n (m ai )2
2 Assume ABC is an isosceles triangle that AB = AC Suppose P is a point on extension of
side BC. X and Y are points on AB and AC that:
P X||AC , P Y ||AB
Also T is midpoint of arc BC. Prove that P T XY
3 Suppose there are 18 lighthouses on the Persian Gulf. Each of the lighthouses lightens an
angle with size 20 degrees. Prove that we can choose the directions of the lighthouses such
that whole of the blue Persian (always Persian) Gulf is lightened.

This file was downloaded from the AoPS Math Olympiad Resources Page
http://www.artofproblemsolving.com/

Page 1

iran
Team Selection Test
2005

Day 2
1 Find all f : N 7 N that there exist k N and a prime p that: n k f (n + p) = f (n) and
also if m | n then f (m + 1) | f (n) + 1
2 Suppose there are n distinct points on plane. There is circle with radius r and center O on
the plane. At least one of the points are in the circle. We do the following instructions. At
each step we move O to the baricenter of the point in the circle. Prove that location of O is
constant after some steps.
3 Suppose S = {1, 2, . . . , n} and n 3. There is f : S k 7 S that if a, b S k and a and b
differ in all of elements then f (a) 6= f (b). Prove that f is a function of one of its elements.

This file was downloaded from the AoPS Math Olympiad Resources Page
http://www.artofproblemsolving.com/

Page 2

iran
Team Selection Test
2006

Day 1
1 Suppose that p is a prime number. Find all natural numbers n such that p|(n) and for all
a such that (a, n) = 1 we have
n|a

(n)
p

2 Suppose n coins are available that their mass is unknown. We have a pair of balances and
every time we can choose an even number of coins and put half of them on one side of the
balance and put another half on the other side, therefore a comparison will be done. Our aim
is determining that the mass of all coins is equal or not. Show that at least n 1 comparisons
are required.
3 Suppose ABC is a triangle with M the midpoint of BC. Suppose that AM intersects the
incircle at K, L. We draw parallel line from K and L to BC and name their second intersection
point with incircle X and Y . Suppose that AX and AY intersect BC at P and Q. Prove
that BP = CQ.

This file was downloaded from the AoPS Math Olympiad Resources Page
http://www.artofproblemsolving.com/

Page 1

iran
Team Selection Test
2006

Day 2
4 Let x1 , x2 , . . . , xn be real numbers. Prove that
n
X
i,j=1

|xi + xj | n

n
X

|xi |

i=1

5 Let ABC be a triangle such that its circumcircle radius is equal to the radius of outer
inscribed circle with respect to A. Suppose that the outer inscribed circle with respect to A
touches BC, AC, AB at M, N, L. Prove that O (Center of circumcircle) is the orthocenter of
M N L.
6 Let G be a tournoment such that its edges are colored either red or blue. Prove that there
exists a vertex of G like v with the property that, for every other vertex u there is a mono-color
directed path from v to u.

This file was downloaded from the AoPS Math Olympiad Resources Page
http://www.artofproblemsolving.com/

Page 2

iran
Team Selection Test
2006

Day 3
1 We have n points in the plane, no three on a line. We call k of them good if they form a
convex polygon and there is no other point in the convex polygon. Suppose that for a fixed
k the number of k good points is ck . Show that the following sum is independent of the
structure of points and only depends on n :
n
X

(1)i ci

i=3

2 Let n be a fixed natural number. a) Find all solutions to the following equation :
n
X
x
[ k] = x1
2
k=1

b) Find the number of solutions to the following equation (m is a fixed natural) :


n
X
x
[ k] = xm
2
k=1

3 Let l, m be two parallel lines in the plane. Let P be a fixed point between them. Let E, F be
variable points on l, m such that the angle EP F is fixed to a number like where 0 < < 2 .
(By angle EP F we mean the directed angle) Show that there is another point (not P ) such
that it sees the segment EF with a fixed angle too.

This file was downloaded from the AoPS Math Olympiad Resources Page
http://www.artofproblemsolving.com/

Page 3

iran
Team Selection Test
2006

Day 4
4 Let n be a fixed natural number. Find all n tuples of natural pairwise distinct and coprime
numbers like a1 , a2 , . . . , an such that for 1 i n we have
a1 + a2 + . . . + an |ai1 + ai2 + . . . + ain
5 Let ABC be an acute angle triangle. Suppose that D, E, F are the feet of perpendicluar lines
from A, B, C to BC, CA, AB. Let P, Q, R be the feet of perpendicular lines from A, B, C to
EF, F D, DE. Prove that
2(P Q + QR + RP ) DE + EF + F D
6 Suppose we have a simple polygon (that is it does not intersect itself, but not necessarily
convex). Show that this polygon has a diameter which is completely inside the polygon and
the two arcs it creates on the polygon perimeter (the two arcs have 2 vertices in common)
both have at least one third of the vertices of the polygon.

This file was downloaded from the AoPS Math Olympiad Resources Page
http://www.artofproblemsolving.com/

Page 4

iran
Team Selection Test
2007

Day 1
1 In triangle ABC, M is midpoint of AC, and D is a point on BC such that DB = DM . We
know that 2BC 2 AC 2 = AB.AC. Prove that
BD.DC =

AC 2 .AB
2(AB + AC)

2 Let A be the largest subset of {1, . . . , n} such that for each x A, x divides at most one
other element in A. Prove that
 
3n
2n
|A|
3
4
edited by pbornsztein
3 Find all solutions of the following functional equation:
f (x2 + y + f (y)) = 2y + f (x)2

This file was downloaded from the AoPS Math Olympiad Resources Page
http://www.artofproblemsolving.com/

Page 1

iran
Team Selection Test
2007

Day 2
1 In an isosceles right-angled triangle shaped billiards table , a ball starts moving from one of
the vertices adjacent to hypotenuse. When it reaches to one side then it will reflect its path.
Prove that if we reach to a vertex then it is not the vertex at initial position
By Sam Nariman
2 Find all monic polynomials f (x) in Z[x] such that f (Z) is closed under multiplication.
By Mohsen Jamali
3 Let be incircle of ABC. P and Q are on AB and AC, such that P Q is parallel to BC
and is tangent to . AB, AC touch at F, E. Prove that if M is midpoint of P Q, and T is
intersection point of EF and BC, then T M is tangent to .
By Ali Khezeli

This file was downloaded from the AoPS Math Olympiad Resources Page
http://www.artofproblemsolving.com/

Page 2

iran
Team Selection Test
2007

Day 3
1 1) Does there exist a a sequence
P a0 , a1 , a2 , . . . in N, such that for each i 6= j, (ai , aj ) = 1, and
for each n, the polynomial ni=0 ai xi is irreducible in Z[x]?
By Omid Hatami
2 Suppose n lines in plane are such that no two are parallel and no three are concurrent. For

each
 two lines their angle is a real number in [0, 2 ]. Find the largest value of the sum of the
n
2 angles between line.
By Aliakbar Daemi
3 O is a point inside triangle ABC such that OA = OB + OC. Suppose B 0 , C 0 be midpoints of
arcs AOC and AOB. Prove that circumcircles COC 0 and BOB 0 are tangent to each other.

This file was downloaded from the AoPS Math Olympiad Resources Page
http://www.artofproblemsolving.com/

Page 3

iran
Team Selection Test
2007

Day 4
1 Find all polynomials of degree 3, such that for each x, y 0:
p(x + y) p(x) + p(y)
2 Triangle ABC is isosceles (AB = AC). From A, we draw a line ` parallel to BC. P, Q are
on perpendicular bisectors of AB, AC such that P Q BC. M, N are points on ` such that
angles AP M and AQN are 2 . Prove that
1
2
1
+

AM
AN
AB
3 Let P be a point in a square whose side are mirror. A ray of light comes from P and with slope
. We know that this ray of light never arrives to a vertex. We make an infinite sequence of
0, 1. After each contact of light ray with a horizontal side, we put 0, and after each contact
with a vertical side, we put 1. For each n 1, let Bn be set of all blocks of length n, in this
sequence. a) Prove that Bn does not depend on location of P . b) Prove that if is irrational,
then |Bn | = n + 1.

This file was downloaded from the AoPS Math Olympiad Resources Page
http://www.artofproblemsolving.com/

Page 4

iran
Team Selection Test
2008

1 Find all functions f : R R such that for each x, y R:


f (xf (y)) + y + f (x) = f (x + f (y)) + yf (x)
2 Suppose that I is incenter of triangle ABC and l0 is a line tangent to the incircle. Let l be
another line such that intersects AB, AC, BC respectively at C 0 , B 0 , A0 . We draw a tangent
from A0 to the incircle other than BC, and this line intersects with l0 at A1 . B1 , C1 are
similarly defined. Prove that AA1 , BB1 , CC1 are concurrent.
3 Suppose that T is a tree with k edges. Prove that the k-dimensional cube can be partitioned
to graphs isomorphic to T .
P
1
takes its minimum
4 Let P1 , P2 , P3 , P4 be points on the unit sphere. Prove that i6=j |Pi P
j|
value if and only if these four points are vertices of a regular pyramid.
5 Let a, b, c > 0 and ab + ac + bc = 1. Prove that:
p
p
p

a3 + a + b3 + b + c3 + c 2 a + b + c
6 Prove that in a tournament with 799 teams, there exist 14 teams, that can be partitioned
into groups in a way that all of the teams in the first group have won all of the teams in the
second group.
7 Let S be a set with n elements, and F be a family of subsets of S with 2n1 elements, such
that for each A, B, C F , A B C is not empty. Prove that the intersection of all of the
elements of F is not empty.
8 Find all polynomials p of one variable with integer coefficients such that if a and b are natural
numbers such that a + b is a perfect square, then p (a) + p (b) is also a perfect square.
9 Ia is the excenter of the triangle ABC with respect to A, and AIa intersects the circumcircle
of ABC at T . Let X be a point on T Ia such that XIa2 = XA.XT . Draw a perpendicular line
from X to BC so that it intersects BC in A0 . Define B 0 and C 0 in the same way. Prove that
AA0 , BB 0 and CC 0 are concurrent.
10 In the triangle ABC, B is greater than C. T is the midpoint of the arc BAC from the
circumcircle of ABC and I is the incenter of ABC. E is a point such that AEI = 90 and
AE k BC. T E intersects the circumcircle of ABC for the second time in P . If B = IP B,
find the angle A.
11 k is a given natural number. Find all functions f : N N such that for each m, n N the
following holds:
f (m) + f (n) | (m + n)k

This file was downloaded from the AoPS Math Olympiad Resources Page
http://www.artofproblemsolving.com/

Page 1

iran
Team Selection Test
2008

12 In the acute-angled triangle ABC, D is the intersection of the altitude passing through A
with BC and Ia is the excenter of the triangle with respect to A. K is a point on the extension
of AB from B, for which AKIa = 90 + 34 C. Ia K intersects the extension of AD at L.
Prove that DIa bisects the angle AIa B iff AL = 2R. (R is the circumradius of ABC)

This file was downloaded from the AoPS Math Olympiad Resources Page
http://www.artofproblemsolving.com/

Page 2

iran
Team Selection Test
2009

Day 1
1 Let ABC be a triangle and A0 , B 0 and C 0 lie on BC , CA and AB respectively such that
the incenter of A0 B 0 C 0 and ABC are coincide and the inradius of A0 B 0 C 0 is half of inradius
of ABC . Prove that ABC is equilateral .
n

2 Let a be a fix natural number . Prove that the set of prime divisors of 22 + a for n = 1, 2,
is infinite
3 Suppose that a,b,c be three positive real numbers such that a + b + c = 3 . Prove that :
1
2+a2 +b2

1
2+b2 +c2

1
2+c2 +a2

3
4

This file was downloaded from the AoPS Math Olympiad Resources Page
http://www.artofproblemsolving.com/

Page 1

iran
Team Selection Test
2009

Day 2
1 Find All Polynomials f with integer coefficient such that , for every prime p and every natural
numbers u and v with the condition : p|uv 1 we always have p|f (u)f (v) 1
2 ABC is a triangle and AA0 , BB 0 and CC 0 are three altitudes of this triangle . Let P be the
feet of perpendicular from C 0 to A0 B 0 , and Q is a point on A0 B 0 such that QA = QB . Prove
that : P BQ = P AQ = P C 0 C
3 We have a closed path on a vertices of a nn square which pass from each vertice exactly once
. prove that we have two adjacent vertices such that if we cut the path from these points
then length of each pieces is not less than quarter of total path .

This file was downloaded from the AoPS Math Olympiad Resources Page
http://www.artofproblemsolving.com/

Page 2

iran
Team Selection Test
2009

Day 3
1 Suppose three direction on the plane . We draw 11 lines in each direction . Find maximum
number of the points on the plane which are on three lines .
2

, (xy)
)
2 Find All Polynomials P (x, y) such that for all reals x, y we have : P (x2 , y 2 ) = P ( (x+y)
2
2
3 In triangle ABC , D , E and F are the points of tangency of incircle with the center of I to
BC , CA and AB respectively . Let M be the feet of perpendicular from D to EF and P
is on DM such that : DP = M P . If H be the orthocenter of BIC, prove that P H bisects
EF .

This file was downloaded from the AoPS Math Olympiad Resources Page
http://www.artofproblemsolving.com/

Page 3

iran
Team Selection Test
2009

Day 4
1 Let ABC be a triangle and AB 6= AC . D is a point on BC such that BA = BD and B is
between C and D . Let Ic be center of the circle which touches AB and the extensions of AC
and BC . CIc intersect the circumcircle of ABC again at T . If T DIc = B+C
then find
4
A
2 n is a positive integer . prove that :
3

n
52 1
2n+2

(5)

n
32 1
2n+2

(mod 2n+4 )

3 T is a subset of 1, 2, ..., n which has this property : for all distinct i, j T , 2j is not divisible
by i . Prove that : |T | 49 n + log2 n + 2

This file was downloaded from the AoPS Math Olympiad Resources Page
http://www.artofproblemsolving.com/

Page 4

iran
Pre-Preparation Course Examination
2004

1 A network is a simple directed graph such that each edge e has two intger lower and upper
capacities 0 cl (e) cu (e). A circular flow on this graph is a function such that: 1) For each
edge e, cl (e) f (e) cu (e). 2) For each vertex v:
X
X
f (e) =
f (e)
ev

ev +

a) Prove that this graph has a circular flow, if and only if for each partition X, Y of vertices
of the network we have:
X
X
cl (e)
cl (e)
e = xy
x X, y Y

e = yx
y Y, x X

b) Suppose that f is a circular flow in this network. Prove that there exists a circular flow g
in this network such that g(e) = bf (e)c or g(e) = df (e)e for each edge e.
2 Let H(n) be the number of simply connected subsets with n hexagons in an infinite hexagonal
network. Also let P (n) be the number of paths starting from a fixed vertex (that do not
connect itself) with lentgh n in this hexagonal network. a) Prove that the limits
1

:= lim H(n) n , := lim P (n) n


n

exist.
b) Prove the following inequalities:

2 2 12.5 3.5 4

3 For a subset S of vertices of graph G, let (S) be the subset of all edges of G such that at least

one of their ends is in S. Suppose


P that G is a graph with m edges. Let d : V (G) N {0}
be a function such that a) u d (u) = m. b) For each subset S of V (G):
X
d (u) |(S)|
uS

Prove that we can give directions to edges of G such that for each edge e, d+ (e) = d (e).
4 Let G be a simple graph. Suppose that size of largest independent set in G is . Prove that:
a) Vertices of G can be partitioned to at most paths. b) Suppose that a vertex and an edge
are also cycles. Prove that vertices of G can be partitioned to at most cycles.
5 Let A = {A1 , . . . , Am } be a family distinct subsets of {1, 2, . . . , n} with at most
Assume that Ai 6 Aj and Ai Aj 6= for each i, j. Prove that:
m
X

i=1

n1
|Ai |1

n
2

elements.

 1

This file was downloaded from the AoPS Math Olympiad Resources Page
http://www.artofproblemsolving.com/

Page 1

iran
Pre-Preparation Course Examination
2004

6 Let l, d, k be natural numbers. We want to prove that for large numbers n, for each k-coloring
of the n-dimensional cube with side length l, there is a d-dimensional subspace that all of its
vertices have the same color. Let H(l, d, k) be the least number such that for n H(l, d, k)
the previus statement holds. a) Prove that:
H(l, d + 1, k) H(l, 1, k) + H(l, d, k l )H(l,1,k)
b) Prove that
H(l + 1, 1, k + 1) H(l, 1 + H(l + 1, 1, k), k + 1)
c) Prove the statement of problem. d) Prove Van der Waerdens Theorem.
7 Let G = (V, E) ve a simple graph.
a) Let A, B be a subsets of E, and spanning subgraphs of G with edges A, B, A B and A B
have a, b, c and d connected components respectively. Prove that a + b c + d
We say that subsets A1 , A2 , . . . , Am of E have (R) property if and only if for each I
{1, 2, . . . , m} the spanning subgraph of G with edges iI Ai has at most n |I| connected
components. b) Prove that when A1 , . . . , Am , B have (R) property, and |B| 2, there exists
an x B such that A1 , A2 , . . . , Am , B\{x} also have property (R).
Suppose that edges of G are colored arbitrarily. A spanning subtree in G is called colorful if
and only if it does not have any two edges with the same color. c) Prove that G has a colorful
subtree if and only if for each partition of V to k non-empty subsets such as V1 , . . . , Vk , there
are at least k 1 edges with distinct colors that each of these edges has its two ends in two
different
Vi s. d) Assume that edges of Kn has been colored such that each color is repeated
n
times.
Prove that there exists a colorful subtree. e) Prove that in part d) if n 5 there
2
is a colorful subtree that is non-isomorphic to K1,n1 . f) Prove that in part e) there are at
least two non-intersecting colorful subtrees.

This file was downloaded from the AoPS Math Olympiad Resources Page
http://www.artofproblemsolving.com/

Page 2

iran
Pre-Preparation Course Examination
2006

Combinatorics

P
(n)
1 a) Find the value of
n=1 2n 1 ;
 n+k P
P
b) Show that k m
= k
k
m
1

m
k

 n
k

2k for m, n 0;

c) Using the identity (1 x) 2 (1 x) 2 = (1 x)1 derive a combinatorial identity!


P
d) Express the value of (2a1 1) . . . (2ak 1) where the sum is over all 2n1 ways of choosing
(a1 , a2 , . . . , ak ) such that a1 + a2 + . . . + ak = n, as a function of some Fibonacci term.
2 If f (x) is the generating function of the sequence a1 , a2 , . . . and if f (x) = r(x)
s(x) holds such that
r(x) and s(x) are polynomials show that an has a homogenous recurrence.
3 The bell number bn is the number of ways to partition the
2, . . . , n}. For example
Pset {1,
n
b3 = 5. Find a recurrence for bn and show that bn = e1 k0 kk! . Using a combinatorial
proof show that the number of ways to partition {1, 2, . . . , n}, such that now two consecutive
numbers are in the same block, is bn1 .
4 Show that for every prime p and integer n, there is an irreducible polynomial of degree n in
Zp [x] and use that to show there is a field of size pn .
5 Express the sum Sm (n) = 1m + 2m + . . . + (n 1)m with Bernolli numbers.
6 Show that the product of every k consecutive members of the Fibonacci sequence is divisible
by f1 f2 . . . fk (where f0 = 0 and f1 = 1).
7 Suppose that for every n the number m(n) is chosen such that m(n) ln(m(n)) = n 12 . Show
that bn is asymptotic to the following expression where bn is the nth Bell number, that is
the number of ways to partition {1, 2, . . . , n}:
1

m(n)n em(n)n 2

.
ln n
Two functions f (n) and g(n) are asymptotic to each other if limn

f (n)
g(n)

= 1.

8 Suppose that p(n) is the number of ways to express n as a sum of some naturall numbers (the
two representations 4 = 1 + 1 + 2 and 4 = 1 + 2 + 1 are considered the same). Prove that for
an infinite number of ns p(n) is even and for an infinite number of ns p(n) is odd.

This file was downloaded from the AoPS Math Olympiad Resources Page
http://www.artofproblemsolving.com/

Page 1

iran
Pre-Preparation Course Examination
2006

Dynamical Systems

1 Suppose that X is a compact metric space and T : X X is a continous function. Prove


that T has a returning point. It means there is a strictly increasing sequence ni such that
limk T nk (x0 ) = x0 for some x0 .
2 Show that there exists a continuos function f : [0, 1] [0, 1] such that it has no periodic orbit
of order 3 but it has a periodic orbit of order 5.
3 Show that if f : [0, 1] [0, 1] is a continous function and it has topological transitivity then
periodic points of f are dense in [0, 1]. Topological transitivity means there for every open
sets U and V there is n > 0 such that f n (U ) V 6= .
4 Show that (f ) changes continously over f . It means for every bijection f : S 1 S 1 and
 > 0 there is > 0 such that if g : S 1 S 1 is a bijection such that k f g k < then
|(f ) (g)| < .
Note that (f ) is the rotatation number of f and k f g k = sup{|f (x) g(x)||x S 1 }.
5 Powers of 2 in base 10 start with 3 or 4 more frequently? What is their state in base 3? First
write down an exact form of the question.
6 Suppose that Pc (z) = z 2 + c. You are familiar with the Mandelbrot set: M = {c
C| limn Pcn (0) 6= }. We know that if c M then the points of the dynamical system
(C, Pc ) that dont converge to are connected and otherwise they are completely disconnected. By seeing the properties of periodic points of Pc prove the following ones:
a) Prove the existance of the heart like shape in the Mandelbrot set. b) Prove the existance
of the large circle next to the heart like shape in the Mandelbrot set.
[img]http://astronomy.swin.edu.au/ pbourke/fractals/mandelbrot/mandel1.gif[/img]

This file was downloaded from the AoPS Math Olympiad Resources Page
http://www.artofproblemsolving.com/

Page 2

iran
Pre-Preparation Course Examination
2006

Geometry

1 Show that for a triangle we have


max{ama , bmb , cmc } sR
where ma denotes the length of median of side BC and s is half of the perimeter of the
triangle.
2 Using projective transformations prove the Pascal theorem (also find where the Pascal line
intersects the circle).
3 There is a right angle whose vertex moves on a fixed circle and one of its sides passes a fixed
point. What is the curve that the other side of the angle is always tangent to it.
4 Find a 3rd degree polynomial whose roots are ra , rb and rc where ra is the radius of the outer
inscribed circle of ABC with respect to A.
5 Suppose is a fixed line and F and F 0 are two points with equal distance from that are on
two sides of . The circle C is with center P and radius mP F where m is a positive number
not equal to 1. The circle C 0 is the circle that P F F 0 is inscribed in it.
a) What is the condition on P such that C and C 0 intersect?
b) If we denote the intersections of C and C 0 to be M and M 0 then what is the locus of M
and M 0 ;
c) Show that C is always tangent to this locus.

This file was downloaded from the AoPS Math Olympiad Resources Page
http://www.artofproblemsolving.com/

Page 3

iran
Pre-Preparation Course Examination
2006

Superior Algebra

1 Find out wich of the following polynomials are irreducible.


a) t4 + 1 over R;
b) t4 + 1 over Q;
c) t3 7t2 + 3t + 3 over Q;
d) t4 + 7 over Z17 ;
e) t3 5 over Z11 ;
f) t6 + 7 over Q(i).
2 a) Show that you can divide an angle to three equal parts using compass and ruler if and
only if the polynomial 4t3 3t cos() is reducible over Q(cos()).
b) Is it always possible to divide an angle into five equal parts?
3 a) If K is a finite extension of the field F and K = F (, ) show that [K : F ] [F () :
F ][F () : F ]
b) If gcd([F () : F ], [F () : F ]) = 1 then does the above inequality always become equality?
c) By giving an example show that if gcd([F () : F ], [F () : F ]) 6= 1 then equality might
happen.

4 If d Q, is there always an C such that n = 1 for some n N and Q( d) Q()?

This file was downloaded from the AoPS Math Olympiad Resources Page
http://www.artofproblemsolving.com/

Page 4

iran
Pre-Preparation Course Examination
2007

Combinatorics

1 a) There is an infinite sequence of 0, 1, like . . . , a1 , a0 , a1 , . . . (i.e. an element of {0, 1}Z ).


At each step we make a new sequence. There is a function f such that for each i, new ai =
f (ai100 , ai99 , . . . , ai+100 ). This operation is mapping F : {0, 1}Z {0, 1}Z . Prove that if
F is 1-1, then it is surjective. b) Is the statement correct if we have an fi for each i?
2 There is a WORD game with the following rules. There are finite number of relations Ui
Vi (Ui , Vi are words). There is are two words A, B. We start from A, and we want to reach
to B. At each step we can change one subword Ui to Vi . Prove that there does not exist an
algorithm that picks up A, B and Ui s,Vi s and decides whether we can reach from A to B or
not.

This file was downloaded from the AoPS Math Olympiad Resources Page
http://www.artofproblemsolving.com/

Page 1

iran
Pre-Preparation Course Examination
2007

Geometry

1 D is an arbitrary point inside triangle ABC, and E is inside triangle BDC. Prove that
SDBC
SEBC

2
(PDBC )
(PEBC )2
2 Let C1 , C2 and C3 be three circles that does not intersect and non of them is inside another. Suppose (L1 , L2 ), (L3 , L4 ) and (L5 , L6 ) be internal common tangents of (C1 , C2 ),
(C1 , C3 ), (C2 , C3 ). Let L1 , L2 , L3 , L4 , L5 , L6 be sides of polygon AC 0 BA0 CB 0 . Prove that
AA0 , BB 0 , CC 0 are concurrent.
3 ABC is an arbitrary triangle. A0 , B 0 , C 0 are midpoints of arcs BC, AC, AB. Sides of triangle
ABC, intersect sides of triangle A0 B 0 C 0 at points P, Q, R, S, T, F . Prove that
SP QRST F
ab + ac + bc
=1
SABC
(a + b + c)2
4 Let (C) and (L) be a circle and a line. P1 , . . . , P2n+1 are odd number of points on (L). A1 is
an arbitrary point on (C). Ak+1 is the intersection point of Ak Pk and (C) (1 k 2n + 1).
Prove that A1 A2n+2 passes through a constant point while A1 varies on (C).

This file was downloaded from the AoPS Math Olympiad Resources Page
http://www.artofproblemsolving.com/

Page 2

iran
Pre-Preparation Course Examination
2007

Number Theory Algebra


1 Let a 2 be a natural number. Prove that

1
n=0 an2

is irrational.

2 Let {A1 , . . . , Ak } be matrices which make a group under matrix multiplication. Suppose
M = A1 + + Ak . Prove that each eigenvalue of M is equal to 0 or k.
3 This question is both combinatorics and Number Theory : a ) Prove that we can color edges
of Kp with p colors which is proper, (p is an odd prime) and Kp can be partitioned to p1
2
rainbow Hamiltonian cycles. (A Hamiltonian cycle is a cycle that passes from all of verteces,
and a rainbow is a subgraph that all of its edges have different colors.) b) Find all answers
of x2 + y 2 + z 2 = 1 is Zp
4 a, b Z and for every n N0 , the number 2n a + b is a perfect square. Prove that a = 0.
5 Prove that the equation
y 3 = x2 + 5
doesnt have any solutions in Z.
6 Let a, b be two positive integers and b2 + a 1|a2 + b 1. Prove that b2 + a 1 has at least
two prime divisors.
7 Let p be a prime such that p 3 (mod 4). Prove that we cant partition the numbers
a, a + 1, a + 2, , a + p 2,(a Z) in two sets such that product of members of the sets be
equal.

8 Let m, n, k be positive integers and 1 + m + n 3 = (2 + 3)2k+1 . Prove that m is a perfect


square.
9 Solve the equation 4xy x y = z 2 in positive integers.
10 Let a > 1 be a positive integer. Prove that the set {a2 + a 1, a3 + a 1, } have a subset
S with infinite members and for any two members of S like x, y we have gcd(x, y) = 1. Then
prove that the set of primes has infinite members.
11 Let p 3 be a prime and a1 , a2 , , ap2 be a sequence of positive integers such that for
every k {1, 2, , p 2} neither ak nor akk 1 is divisible by p. Prove that product of some
of members of this sequence is equivalent to 2 modulo p.
12 Find all subsets of N like S such that
m, n S =

m+n
S
gcd(m, n)

This file was downloaded from the AoPS Math Olympiad Resources Page
http://www.artofproblemsolving.com/

Page 3

iran
Pre-Preparation Course Examination
2007

13 Let {ai }
i=1 be a sequence of positive integers such that a1 < a2 < a3 and all of primes
are members of this sequence. Prove that for every n < m
1
1
1
+
+ +
6 N
an an+1
am
14 Find all a, b, c N such that
a2 b|a3 + b3 + c3 ,

b2 c|a3 + b3 + c3 ,

c2 a|a3 + b3 + c3 .

[PS: The original problem was this: Find all a, b, c N such that
a2 b|a3 + b3 + c3 ,

b2 c|a3 + b3 + c3 ,

c2 b|a3 + b3 + c3 .

But I think the author meant c2 a|a3 + b3 + c3 , just because of symmetry]


15 Does there exists a subset of positive integers with infinite members such that for every two
members a, b of this set
a2 ab + b2 |(ab)2
n

16 Prove that 22 + 22

n1

+ 1 has at least n distinct prime divisors.

17 For a positive integer n, denote rad(n) as product of prime divisors of n. And also rad(1) = 1.
Define the sequence {ai }
i=1 in this way: a1 N and for every n N, an+1 = an + rad(an ).
Prove that for every N N, there exist N consecutive terms of this sequence which are in an
arithmetic progression.
18 Prove that the equation x3 + y 3 + z 3 = t4 has infinitely many solutions in positive integers
such that gcd(x, y, z, t) = 1.
19 Find all functions f : N N such that:
i) f 2000 (m) = f (m).
f (m)f (n)
ii) f (mn) =
f (gcd(m, n))
iii) f (m) = 1 m = 1
20 Let m, n be two positive integers and m 2. We know that for every positive integer a such
that gcd(a, n) = 1 we have n|am 1. Prove that n 4m(2m 1).
21 Find all primes p, q such that
pq q p = pq 2 19
22 Prove that for any positive integer n 3 there exist positive integers a1 , a2 , , an such that
a1 a2 an ai

(mod a2i )

i {1, 2, , n}

This file was downloaded from the AoPS Math Olympiad Resources Page
http://www.artofproblemsolving.com/

Page 4

iran
Pre-Preparation Course Examination
2007

Problem Solving Exam

1 a) Find all multiplicative functions f : Zp Zp (i.e. that x, y Zp , f (xy) = f (x)f (y).)


b) How many bijective multiplicative does exist on Zp c) Let A be set of all multiplicative
functions on Zp , and V B be set of all bijective multiplicative functions on Zp . For each
x Zp , calculate the following sums :
X
f A

f (x),

f (x)

f B

2 a) Prove that center of smallest sphere containing a finite subset of Rn is inside convex hull
of the point that lie on sphere. b) A is a finite subset of Rn , and distance of every two points
of A is not larger than 1. Find radius of the largest sphere containing A.
3 Prove that for each a N, there are infinitely many natural n, such that
n | ana+1 1.
4 Prove that
p
p
2007
X
|i + 1|
|i|

>
|i|
( 2)|i|
( 2)
i=2007
i=2007
2007
X

This file was downloaded from the AoPS Math Olympiad Resources Page
http://www.artofproblemsolving.com/

Page 5

iran
Pre-Preparation Course Examination
2008

1 Rk (m, n) is the least number such that for each coloring of k-subsets of {1, 2, . . . , Rk (m, n)}
with blue and red colors, there is a subset with m elements such that all of its k-subsets
are red or there is a subset with n elements such that all of its k-subsets are blue. a) If we
give a direction randomly to all edges of a graph Kn then what is the probability that the
resultant graph does not have directed triangles? b) Prove that there exists a c such that
R3 (4, n) 2cn .
2 Seven points are selected randomly from S 1 C. What is the probability that origin is not
contained in convex hull of these points?
3 Prove that we can put ( 1 ) points on surface of a sphere with radius 1 such that distance of
each of these points and the plane passing through center and two of other points is at least
.
4 Sarah and Darah play the following game. Sarah puts n coins numbered with 1, . . . , n on a
table (Each coin is in HEAD or TAIL position.) At each step Darah gives a coin to Sarah
and she (Sarah) let him (Dara) to change the position of all coins with number multiple of
a desired number k. At the end, all of the coins that are in TAIL position will be given to
Sarah and all of the coins with HEAD position will be given to Darah. Prove that Sarah can
put the coins in a position at the beginning of the game such that she gains at least (n)
coins. [hide=Hint:]Chernov inequality!
5 A permutation is selected randomly through all n-permutations. a) if
Ca () = the number of cycles of length a in
then prove that E(Ca ()) = a1 b) Prove that if {a1 , a2 , . . . , ak } {1, 2, . . . , n} the probability
that does not have any cycle with lengths a1 , . . . , ak is at most Pk1
i=1

This file was downloaded from the AoPS Math Olympiad Resources Page
http://www.artofproblemsolving.com/

ai

Page 1

Vous aimerez peut-être aussi